首页 1994—2001年考研英语真题(附作文解析)1[新版]

1994—2001年考研英语真题(附作文解析)1[新版]

举报
开通vip

1994—2001年考研英语真题(附作文解析)1[新版]1994—2001年考研英语真题(附作文解析)1[新版] 1994 年试题与分析 Section ?Use of English The first and smallest unit that can be discussed in relation to language is the word. In speaking, the choice of words is 1 the utmost importance. Proper selection will eliminate one source of...

1994—2001年考研英语真题(附作文解析)1[新版]
1994—2001年考研英语真题(附作文解析)1[新版] 1994 年试题与分析 Section ?Use of English The first and smallest unit that can be discussed in relation to language is the word. In speaking, the choice of words is 1 the utmost importance. Proper selection will eliminate one source of 2 breakdown is in the communication cycle. Too often, careless use of words__3 a meeting of the minds of the speaker and listener. The words used by the speaker may ___4 unfavorable reactions in the listener 5 interfere with his comprehension; hence, the transmission-reception system breaks down. 6__, inaccurate or indefinite words may make ___7 difficult for the listener to understand the 8 which is being transmitted to him. The speaker who does not have specific words in his working vocabulary may be 9 to explain or describe in a 10 that can be understood by his listeners. 1. [A] of [B] at [C] for [D] on 2.[A] inaccessible [B] timely [C] likely [D] invalid 3.[A] encourages [B] prevents [C] destroys [D] offers 4.[A] pass out [B] take away [C] back up [D] stir up 5.[A] who [B] as [C] which [D] what 6.[A] Moreover [B] However [C] Preliminarily [D] Unexpectedly 7.[A] that [B] it [C] so [D] this 8.[A] speech [B] sense [C] message [D] meaning 9.[A] obscure [B] difficult [C] impossible [D] unable 10.[A] case [B] means [C] method [D] way Section ?Reading Comprehension Passage 1 The American economic system is organized around a basically private-enterprise, market- oriented economy in which consumers largely determine what shall be produced by spending their money in the marketplace for those goods and services that they want most. Private businessmen, striving to make profits, produce these goods and services in competition with other businessmen; and the profit motive, operating under competitive pressures, largely determines how these goods and services are produced. Thus, in the American economic system it is the demand of individual consumers, coupled with the desire of businessmen to maximize profits and the desire of individuals to maximize their incomes, that together determine what shall be produced and how resources are used to produce it. An important factor in a market-oriented economy is the mechanism by which consumer demands can be expressed and responded to by producers. In the American economy, this mechanism is provided by a price system, a process in which prices rise and fall in response to relative demands of consumers and supplies offered by seller-producers. If the product is in short supply relative to the demand, the price will be bid up and some consumers will be eliminated from the market. If, on the other hand, producing more of a commodity results in reducing its cost, this will tend to increase the supply offered by seller-producers, which in turn will lower the price and permit more consumers to buy the product. Thus, price is the regulating mechanism in the American economic system. The important factor in a private enterprise economy is that individuals are allowed to own productive resources (private property) , and they are permitted to hire labor, gain control over natural resources, and produce goods and services for sale at a profit. In the American economy, the concept of private property embraces not only the ownership of productive resources but also certain rights, including the right to determine the price of a product or to make a free contract with another private individual. 11. In Line 11, Para 1, “the desire of individuals to maximize their incomes” means__. [A] Americans are never satisfied with their incomes [B] Americans tend to overstate their incomes [C] Americans want to have their incomes increased [D] Americans want to increase the purchasing power of their incomes 12. The first two sentences in the second paragraph tell us that__. [A]producers can satisfy the consumers by mechanized production [B]consumers can express their demands through producers [C]producers decide the prices of products [D]supply and demand regulate prices 13. According to the passage, a private-enterprise economy is characterized by__. [A]private property and rights concerned [B]manpower and natural resources control [C]ownership of productive resources [D]free contracts and prices 14. The passage is mainly about__. [A]how American goods are produced [B]how American consumers buy their goods [C]how American economic system works [D]how American businessmen make their profits Passage 2 One hundred and thirteen million Americans have at least one bank-issued credit card. They give their owners automatic credit in stores, restaurants, and hotels, at home, across the country, and even abroad, and they make many banking services available as well. More and more of these credit cards can be read automatically, making it possible to withdraw or deposit money in scattered locations, whether or not the local branch bank is open. For many of us the “cashless society” is not on the horizon—it?s already here. While computers offer these conveniences to consumers, they have many advantages for sellers too. Electronic cash registers can do much more than simply ring up sales. They can keep a wide range of records, including who sold what, when, and to whom. This information allows businessmen to keep track of their list of goods by showing which items are being sold and how fast they are moving. Decisions to reorder or return goods to suppliers can then be made. At the same time these computers record which hours are busiest and which employees are the most efficient, allowing personnel and staffing assignments to be made accordingly. And they also identify preferred customers for promotional. Computers are relied on by manufacturers for similar reasons. Computer-analyzed marketing reports can help to decide which products to emphasize now, which to develop for the future, and which to drop. Computers keep track of goods in stock, of raw materials on hand, and even of the production process itself. Numerous other commercial enterprises, from theaters to magazine publishers, from gas and electric utilities to milk processors, bring better and more efficient services to consumers through the use of computers. 15. According to the passage, the credit card enables its owner to__. [A]withdraw as much money from the bank as he wishes [B]obtain more convenient services than other people do [C]enjoy greater trust from the storekeeper [D]cash money wherever he wishes to 16. From the last sentence of the first paragraph we learn that __. [A]in the future all the Americans will use credit cards [B]credit cards are mainly used in the United States today [C]nowadays many Americans do not pay in cash [D]it is now more convenient to use credit cards than before 17. The phrase "ring up sales" (Line 3, Para. 2) most probably means “___”. [A]make an order of goods [B] record sales on a cash register [C]call the sales manager [D] keep track of the goods in stock 18. What is this passage mainly about? [A] Approaches to the commercial use of computers. [B] Conveniences brought about by computers in business. [C] Significance of automation in commercial enterprises. [D] Advantages of credit cards in business. Passage 3 Exceptional children are different in some significant way from others of the same age. For these children to develop to their full adult potential, their education must be adapted to those differences. Although we focus on the needs of exceptional children, we find ourselves describing their environment as well. While the leading actor on the stage captures our attention, we are aware of the importance of the supporting players and the scenery of the play itself. Both the family and the society in which exceptional children live are often the key to their growth and development. And it is in the public schools that we find the full expression of society? s understanding—the knowledge, hopes, and fears that are passed on to the next generation. Education in any society is a mirror of that society. In that minor we can see the strengths, the weaknesses, the hopes, the prejudices, and the central values of the culture itself. The great interest in exceptional children shown in public education over the past three decades indicates the strong feeling in our society that all citizens, whatever their special conditions, deserve the opportunity to fully develop their capabilities. “All men are created equal.” We?ve heard it many times, but it still has important meaning for education in a democratic society. Although the phrase was used by this country?s founders to denote equality before the law, it has also been interpreted to mean equality of opportunity. That concept implies educational opportunity for all children—the right of each child to receive help in learning to the limits of his or her capacity, whether that capacity be small or great. Recent court decisions have confirmed the right of all children—disabled or not—to an appropriate education, and have ordered that public schools take the necessary steps to provide that education. In response, schools are modifying their programs, adapting instruction to children who are exceptional, to those who cannot profit substantially from regular programs. 19. In Paragraph 2, the author cites the example of the leading actor on the stage to show that _____. [A]the growth of exceptional children has much to do with their family and the society [B]exceptional children are more influenced by their families than normal children are [C]exceptional children are the key interest of the family and society [D]the needs of the society weigh much heavier than the needs of the exceptional children 20. The reason that the exceptional children receive so much concern in education is that__. [A]they are expected to be leaders of the society [B]they might become a burden of the society [C]they should fully develop their potentials [D]disabled children deserve special consideration 21. This passage mainly deals with__. [A]the differences of children in their learning capabilities [B]the definition of exceptional children in modern society [C]the special educational programs for exceptional children [D]the necessity of adapting education to exceptional children 22. From this passage we learn that the educational concern for exceptional children ___. [A]is now enjoying legal support [B]disagrees with the tradition of the country [C]was clearly stated by the country? s founders [D]will exert great influence over court decisions Passage 4 “I have great confidence that by the end of the decade we?ll know in vast detail how cancer cells arise,” says microbiologist Robert Weinberg, an expert on cancer. “But,” he cautions, “Some people have the idea that once one understands the causes, the cure will rapidly follow. Consider Pasteur. He discovered the causes of many kinds of infections, but it was fifty or sixty years before cures were available. ” This year, 50 percent of the 910,000 people who suffer from cancer will survive at least five years. In the year 2000, the National Cancer Institute estimates, that figure will be 75 percent. For some skin cancers, the five-year survival rate is as high as 90 percent. But other survival statistics are still discouraging—13 percent for lung cancer, and 2 percent for cancer of the pancreas (胰腺) . With as many as 120 varieties in existence, discovering how cancer works is not easy. The researchers made great progress in the early 1970s, when they discovered that oncogenes, which are cancer-causing genes (基因), are inactive in normal cells. Anything from cosmic rays to radiation to diet may activate a dormant oncogene, but how remains unknown. If several oncogenes are driven into action, the cell, unable to turn them off, becomes cancerous. The exact mechanisms involved are still mysterious, but the likelihood that many cancers are initiated at the level of genes suggests that we will never prevent all cancers. “Changes are a normal part of the evolutionary process, “says oncologist William Haywar. Environmental factors can never be totally eliminated; as Hayward points out, “We can?t prepare a medicine against cosmic rays.” The prospects for cure, though still distant, are brighter. "First, we need to understand how the normal cell controls itself. Second, we have to determine whether there are a limited number of genes in cells which are always responsible for at least part of the trouble. If we can understand how cancer works, we can counteract its action. " 23. The example of Pasteur in the passage is used to__. [A]. predict that the secret of cancer will be disclosed in a decade [B]indicate that the prospects for curing cancer are bright [C]prove that cancer will be cured in fifty to sixty years [D]warn that there is still a long way to go before cancer can be conquered 24. The author implies that by the year 2000, __. [A]there will be a drastic rise in the five-year survival rate of skin-cancer patients [B] 90 percent of the skin-cancer patients today will still be living [C]the survival statistics will be fairly even among patients with various cancers [D]there won? t be a drastic increase of survival rate of all cancer patients 25. Oncogenes are cancer-causing genes__. [A]that are always in operation in a healthy person [B]which remain unharmful so long as they are not activated [C]that can be driven out of normal cells [D]which normal cells can? t turn off 26. The word "dormant" in the third paragraph most probably means__. [A]dead [B] ever-present [C] inactive [D] potential Passage 5 Discoveries in science and technology are thought by “untaught minds” to come in blinding flashes or as the result of dramatic accidents. Sir Alexander Fleming did not, as legend would have it, look at the mold (霉) on a piece of cheese and get the idea for penicillin there and then. He experimented with antibacterial substances for nine years before he made his discovery. Inventions and innovations almost always come out of laborious trial and error. Innovation is like soccer; even the best players miss the goal and have their shots blocked much more frequently than they score. The point is that the players who score most are the ones who take most shots at the goal—and so it goes with innovation in any field of activity. The prime difference between innovation and others is one of approach. Everybody gets ideas, but innovators work consciously on their sand they follow them through until they prove practicable or otherwise. What ordinary people see as fanciful abstractions, professional innovators see as solid possibilities. "Creative thinking may mean simply the realization that there? s no particular virtue in doing things the way they have always been done, " wrote Rudolph Flesch, a language authority. This accounts for our reaction to seemingly simple innovations like plastic garbage bags and suitcases on wheels that make life more convenient: "How come nobody thought of that before?" The creative approach begins with the proposition that nothing is as it appears. Innovators will not accept that there is only one way to do anything. Faced with getting from A to B, the average person will automatically set out on the best-known and apparently simplest route. The innovator will search for alternate courses, which may prove easier in the long run and are bound tobe more interesting and challenging even if they lead to dead ends. Highly creative individuals really do march to a different drummer. 27. What does the author probably mean by "untaught mind" in the first paragraph? [A] A person ignorant of the hard work involved in experimentation. [B] A citizen of a society that restricts personal creativity. [C] A person who has had no education. [D] An individual who often comes up with new ideas by accident. 28. According to the author, what distinguishes innovators from non-innovators? [A] The variety of ideas they have. [B] The intelligence they possess. [C] The way they deal with problems. [D] The way they present their findings. 29. The author quotes Rudolph Flesch in Paragraph 3 because__. [A] Rudolph Flesch is the best-known expert in the study of human creativity [B]the quotation strengthens the assertion that creative individuals look for new ways of doing things . [C]the reader is familiar with Rudolph Flesch? s point of view [D]the quotation adds a new idea to the information previously presented 30. The phrase “march to a different drummer” (the last line of the passage) suggests that highly creative individuals are__. [A]diligent in pursuing their goals [B]reluctant to follow common ways of doing things [C]devoted to the progress of science [D]concerned about the advance of society Part ? English-Chinese Translation According to the new school of scientists, technology is an overlooked force in expanding the horizons of scientific knowledge. (31) Science moves forward, they say, not so much through the insights of great men of genius as because of more ordinary things like improved techniques and tools. (32) “In short”, a leader of the new school contends, “the scientific revolution, as we call it, was largely the improvement and invention and use of a series of instruments that expanded the reach of science in innumerable directions.” (33)Over the years, tools and technology themselves as a source of fundamental innovation have largely been ignored by historians and philosophers of science. The modern school that hails technology argues that such masters as Galileo, Newton, Maxwell, Einstein, and inventors such as Edison attached great importance to, and derived great benefit from, craft information and technological devices of different kinds that were usable in scientific experiments. The centerpiece of the argument of a technology-yes, genius-no advocate was an analysis of Galileo?s role at the start of the scientific revolution. The wisdom of the day was derived from Ptolemy, an astronomer of the second century, whose elaborate system of the sky put Earth at the center of all heavenly motions. (34) Galileo?s greatest glory was that in 1609 he was the first person to turn the newly invented telescope on the heavens to prove that the planets revolve around the sun rather than around the Earth. But the real hero of the story, according to the new school of scientists, was the long evolution in the improvement of machinery for making eye-glasses. Federal policy is necessarily involved in the technology vs. genius dispute. (35)Whether the Government should increase the financing of pure science at the expense of technology or vice versa (反之) often depends on the issue of which is seen as the driving force. Part ? Writing (15 points) DIRECTIONS: A. Title: ON MAKING FRIENDS B. TIME LIMIT: 40 minutes C. Word limit: 120 - 150 words (not including the given opening sentence) D. Your composition should be based on the OUTLINE below and should start with the given opening sentence: “As a human being, one can hardly do without a friend.” E. Your composition must be written clearly on the ANSWER SHEET. OUTLINE: l. The need for friends 2. True friendship 3. My principle in making friends 一、审题 本文题目是我们较为常见的关于交友的议论文。虽然关于友谊的文章很多,但要结构清晰仍需要解析友谊 的真正内涵,指出友谊就是彼此帮助,互相促进学习。 二、谋篇 本文应按提纲所示分为三段:第一段为朋友的重要性。第二段解释友谊的含义。第三段说明自己的择友观。 本文虽条理比较明了,但应注意呼应。 三、写作误区 1.跑题:本文虽与友谊有关,但不是单纯赞扬友谊的。有的考生忽略了“交友”,而把笔墨浪费在赞扬友 谊的珍贵上,因此被扣分也在情理之中。 2.谋篇:本文应以说明议论为主,如果过多地举例描写,又无适当回扣主题,就会出现结构上的偏差,或 者字数过多。 四、范文 评分标准1 15~13分: 内容 财务内部控制制度的内容财务内部控制制度的内容人员招聘与配置的内容项目成本控制的内容消防安全演练内容 切题,包括提纲的全部要点,表达清楚,文字连贯,句式有变化,句子结构和用词正确。 Sample 1 (14 points) On Making Friends As a human being, one can hardly do without a friend and the fact has been self-evident. Without the help of honest friends, we can hardly get over the difficulties and hardships we come across. Moreover, having no friends also means that there is nobody in the world with whom we can share our joy and sorrow. Thus loneliness is the only ting left for us. So making friends is a very indispensable part in our life. But not all who are friendly to each other and enjoy each other?s company, in the social gatherings can be called true friends. Based on mutual understanding instead of on mutual benefit, true friendship must be sincere and unconditional. It consists in sincerity, mutual trust, and self-sacrifice, both in times of happiness and at moments of crisis. A true friend does not just tell us what we want to hear but will do what he thinks is best for us in the long run. False friendship, however, will flatter us when we are crowned with success while desert us when we are in deep waters. Therefore, true friendship is to make people help each other and learn from each other. My principle in making friends is to make as many good friends as possible. So when I make a friend, I first of all make sure whether he is honest or wicked. As long as he is honest and sincere, I know that I find another a company in the way of improving. 例文分析 1(提纲 ,1,论证并得出结论:朋友为生活中不可缺少的一别分, ,2,朋友的真正含义:真诚,互助互勉, ,3,交友之道:真心,真诚。 2(论点与谋篇 本文以提纲为依据,层次清晰。首段采用先解释首句,而后自然引出论题——交友的方式,内容自然流畅。 第二段的优点在于以总—分—总的方法写作,并采用正反论证,条理清晰。第三段直接点明交友之道。 3(连贯性 本文连词使用广泛,比如:第二段末Therefore这一连词的应用,起到收尾的作用,自然引出该段的结论, 使行文连贯。倒数第二句中while的使用,使句子前后既形成了鲜明的对比,又过渡自然。又如第三段的so 使句子间因果关系更为明了,as long as顺接上文,并再次强调中心。 4(句式及用词 句式变化多样是本文另一特点,在文中已标出。 评分标准2 12~10分:内容切题,包括提纲的全部要点,表达比较清楚,文字基本连贯,句式有一定变化,句子结构 和用词无重大错误。 Sample 2 (10 points) On Making Friends As a human being, one can hardly do without a friend. We need friends to share our happiness and sad. We go to friends for help when we are in trouble. We consult with friends if there are problems we cannot handle alone. In a sense, friends are often more important to us than relatives. As the proverb goes, a friend in need is a friend indeed. True friend means when you are in good condition, they are happy for you, and hope you could be better. When you are in trouble, they help you without complain. Also, they should treat you sincerely and not cheat you. When I choose friends, I do not care what work they do or where they are from, but I care those little things which shows one?s characters. I would make sure that we really care for each other. Sometimes we cannot see each other often. But I know my friends will always be there, ready to help if I need them. And I know we are true friends indeed. 例文分析 1(论点与谋篇 题目的中心得到贯穿。结尾ready to help if I need them与we are true friends indeed既呼应全文,又暗指谚语 a friend in need is a friend indeed,语意准确。 2(连贯性与用词 第二段的人称与其他段不一致,如能相同,语意更连贯。使用了一定的连词。短文中用词有一些错误,如: 典型错误:动词complain的名词形式为complaint。 1995年试题与分析 Section ?Use of English Sleep is divided into periods of so-called REM sleep, characterized by rapid eye movements and dreaming, and longer periods of non-REM sleep. 1 kind of sleep is at all well-understood, but REM sleep is 2 to serve some restorative function of the brain. The purpose of non-REM sleep is even more3 . The new experiments, such as these 4 for the first time at a recent meeting of the Society for Sleep Research in Minneapolis, suggest fascinating explanations 5 of non-REM sleep. For example, it has long been known that total sleep 6 is 100 percent fatal to rats, yet, 7 _examinations of the dead bodies, the animals look completely normal. A researcher has now 8 the mystery of why the animals die. The rats 9 bacterial infections of the blood, 10 their immune systems—the self-protecting mechanisrn against disease—had crashed. 1. [A] Either [B] Neither [C] Each [D] Any 2. [A] intended [B] required [C] assumed [D] inferred 3. [A] subtle [B] obvious [C] mysterious [D] doubtful 4. [A] maintained [B] described [C] settled [D] afforded 5. [A] in the light [B] by virtue [C] with the exception [D] for the purpose 6. [A] reduction [B] destruction [C] deprivation [D] restriction 7. [A] upon [B] by [C] through [D] with 8. [A] paid attention to [B] caught sight of [C] laid emphasis on [D] cast light on 9. [A] develop [B] produce [C] stimulate [D] induce 10. [A] if [B] as if [C] only if [D] if only Section ?Reading Comprehension Passage l Money spent on advertising is money spent as well as any I know of. It serves directly to assist a rapid distribution of goods at reasonable price, thereby establishing a firm home market and so making it possible to provide for export at competitive prices. By drawing attention to new ideas it helps enormously to raise standards of living. By helping to increase demand it ensures an increased need for labor, and is therefore an effective way to fight unemployment. It lowers the costs of many services: without advertisements your daily newspaper would cost four times as much, the price of your television licence would need to be doubled, and travel by bus or tube would cost 20 per cent more. And perhaps most important of all, advertising provides a guarantee of reasonable value in the products and services you buy. Apart from the fact that twenty-seven Acts of Parliament govern the terms of advertising, no regular advertiser dare promote a product that fails to live up to the promise of his advertisements. He might fool some people for a little while through misleading advertising. He will not do so for long, for mercifully the public has the good sense not to buy the inferior article more than once. If you see an article consistently advertised, it is the surest proof I know that the article does what is claimed for it, and that it represents good value. Advertising does more for the material benefit of the community than any other force I can think of. There is one more point I feel I ought to touch on. Recently I heard a well-known television personality declare that he was against advertising because it persuades rather than informs. He was drawing excessively fine distinctions. Of course advertising seeks to persuade. If its message were confined merely to information—and that in itself would be difficult if not impossible to achieve, for even a detail such as the choice of the colour of a shirt is subtly persuasive—advertising would be so boring that no one would pay any attention. But perhaps that is what the well-known television personality wants. 11. By the first sentence of the passage the author means that__. [A]he is fairly familiar with the cost of advertising [B]everybody knows well that advertising is money consuming [C]advertising costs money like everything else [D]it is worthwhile to spend money on advertising 12. In the passage, which of the following is NOT included in the advantages of advertising? [A] Securing greater fame. [B] Providing more jobs. [C] Enhancing living standards. [D] Reducing newspaper cost. 13. The author deems that the well-known TV personality is_. [A]very precise in passing his judgment on advertising [B]interested in nothing but the buyers' attention [C]correct in telling the difference between persuasion and information [D]obviously partial in his views on advertising 14. In the author's opinion,__. [A]advertising can seldom bring material benefit to man by providing information [B]advertising informs people of new ideas rather than wins them over [C]there is nothing wrong with advertising in persuading the buyer [D]the buyer is not interested in getting information from an advertisement Passage 2 There are two basic ways to see growth: one as a product, the other as a process. People have generally viewed personal growth as an external result or product that can easily be identified and measured. The worker who gets a promotion, the student whose grades improve, the foreigner who learns a new language—all these are examples of people who have measurable results to show for their efforts. By contrast, the process of personal growth is much more difficult to determine, since by definition it is a journey and not the specific signposts or landmarks along the way. The process is not the road itself, but rather the attitudes and feelings people have, their caution or courage, as they encounter new experiences and unexpected obstacles. In this process, the journey never really ends; there are always new ways to experience the world, new ideas to try, new challenges to accept. In order to grow, to travel new roads, people need to have a willingness to take risks, to confront the unknown, and to accept the possibility that they may “fail” at first. How we see ourselves as we try a new way of being is essential to our ability to grow. Do we perceive ourselves as quick and curious? If so, then we tend to take more chances and to be more open to unfamiliar experiences. Do we think we're shy and indecisive? Then our sense of timidity can cause us to hesitate, to move slowly, and not to take a step until we know the ground is safe. Do we think we?re slow to adapt to change or that we?re not smart enough to cope with a new challenge? Then we are likely to take a more passive role or not try at all. These feelings of insecurity and self-doubt are both unavoidable and necessary if we are to change and grow. If we do not confront and overcome these internal fears and doubts, if we protect ourselves too much, then we cease to grow. We become trapped inside a shell of our own making. 15 . A person is generally believed to achieve personal growth when____. [A]he has given up his smoking habit [B]he has made great efforts in his work [C]he is keen on learning anything new [D]he has tried to determine where he is on his journey 16. In the author?s eyes, one who views personal growth as a process would____. [A]succeed in climbing up the social ladder [B]judge his ability to grow from his own achievements [C]face difficulties and take up challenges [D]aim high and reach his goal each time 17. When the author says "a new way of being" (line 3, Para. 3) he is referring to__. [A]a new approach to experiencing the world [B]a new way of taking risks [C]a new method of perceiving ourselves [D]a new system of adaptation to change 18. For personal growth, the author advocates all of the following except_____. [A]curiosity about more chances [B]promptness in self-adaptation [C]open-mindedness to new experiences [D]avoidance of internal fears and doubts Passage 3 In such a changing, complex society formerly simple solutions to informational needs become complicated. Many of life?s problems which were solved by asking family members, friends or colleagues are beyond the capability of the extended family to resolve. Where to turn for expert information and how to determine which expert advice to accept are questions facing many people today. In addition to this, there is the growing mobility of people since World War ?. As families move away from their stable community, their friends of many years, their extended family relationships, the informal flow of information is cut off, and with it the confidence that information will be available when needed and will be trustworthy and reliable. The almost unconscious flow of information about the simplest aspects of living can be cut off. Thus, things once learned subconsciously through the casual communications of the extended family must be consciously learned. Adding to societal changes today is an enormous stockpile of information. The individual now has more information available than any generation, and the task of finding that one piece of information relevant to his or her specific problem is complicated, time-consuming and sometimes even overwhelming . Coupled with the growing quantity of information is the development of technologies which enable the storage and delivery of more information with greater speed to more locations than has ever been possible before. Computer technology makes it possible to store vast amounts of data in machine-readable files, and to program computers to locate specific information. Telecommunications developments enable the sending of messages via television, radio, and very shortly, electronic mail to bombard people with multitudes of messages. Satellites have extended the power of communications to report events at the instant of occurrence. Expertise can be shared world wide through teleconferencing, and problems in dispute can be settled without the participants leaving their homes and/or jobs to travel to a distant conference site. Technology has facilitated the sharing of information and the storage and delivery of information, thus making more information available to more people. In this world of change and complexity, the need for information is of greatest importance. Those people who have accurate, reliable up-to-date information to solve the day-to-day problems, the critical problems of their business, social and family life, will survive and succeed. “Knowledge is power” may well be the truest saying and access to information may be the most critical requirement of all people. 19. The word "it" (line 3, Para. 2) most probably refers to__. [A]the lack of stable communities [B]the breakdown of informal information channels [C]the increased mobility of families [D]the growing number of people moving from place to place 20. The main problem people may encounter today arises form the fact that__. [A]they have to learn new things consciously [B]they lack the confidence of securing reliable and trustworthy information [C]they have difficulty obtaining the needed information readily [D]they can hardly carry out casual communications with an extended family. 21 . From the passage we can infer that__. [A]electronic mail will soon play a dominant role in transmitting messages [B]it will become more difficult for people to keep secrets in an information era [C]people will spend less time holding meetings or conferences [D]events will be reported on the spot mainly through satellites 22. We can learn from the last paragraph that __. [A]it is necessary to obtain as much [B]people should make the best use of the information [C]we should realize the importance of accumulating information . [D]it is of vital importance to acquire needed information efficiently Passage 4 Personality is to a large extent inherent—A-type parents usually bring about A-type offspring. But the environment must also have a profound effect, since if competition is important to the parents; it is likely to become a major factor in the lives of their children. One place where children soak up A-characteristics is school, which is, by its very nature, a highly competitive institution. Too many schools adopt the 'win at all costs' moral standard and measure their success by sporting achievements. The current passion for making children compete against their classmates or against the clock produces a two-layer system , in which competitive A-types seem in some way better than their B-type fellows. Being too keen to win can have dangerous consequences: remember that Pheidippides, the first marathon runner, dropped dead seconds after saying: “Rejoice, we conquer!” By far the worst form of competition in schools is the disproportionate emphasis on examinations. It is a rare school that allows pupils to concentrate on those things they do well. The merits of competition by examination are somewhat questionable, but competition in the certain knowledge of failure is positively harmful. Obviously, it is neither practical nor desirable that all A youngsters change into B?s. The world needs types, and schools have an important duty to try to fit a child?s personality to his possible future employment. It is top management. If the preoccupation of schools with academic work was lessened, more time might be spent teaching children surer values. Perhaps selection for the caring professions, especially medicine, could be made less by good grades in chemistry and more by such considerations as sensitivity and sympathy. It is surely a mistake to choose our doctors exclusively from A-type stock. B?s are important and should be encouraged. 23. According to the passage, A-type individuals are usually__. [A] impatient [B] considerate [C] aggressive [D] agreeable 24. The author is strongly opposed to the practice of examinations at schools because__. [A] the pressure is too great on the students [B] some students are bound to fail [C] failure rates are too high [D] the results of exanimations are doubtful 25. The selection of medical professionals are currently based on__. [A] Candidates? sensitivity [B] academic achievements [C] competitive spirit [D] surer values 26. From the passage we can draw the conclusion that__. [A] the personality of a child is well established at birth [B] family influence dominates the shaping of one?s characteristics. [C] the development of one?s personality is due to multiple factors [D] B-type characteristics can find no place in competitive society Passage 5 That experiences influence subsequent behaviour is evidence of an obvious but nevertheless remarkable activity called remembering. Learning could not occur without the function popularly named memory. Constant practice has such as effect on memory as to lead to skillful performance on the piano, to recitation of a poem, and even to reading and understanding these words. So-called intelligent behaviour demands memory, remembering being a primary requirement for reasoning. The ability to solve any problem or even to recognize that a problem exists depends on memory. Typically, the decision to cross a street is based on remembering many earlier experiences. Practice (or review) tends to build and maintain memory for a task or for any learned material. Over a period of no practice what has been learned tends to be forgotten; and the adaptive consequences may not seem obvious. Yet, dramatic instances of sudden forgetting can seem to be adaptive. In this sense, the ability to forget can be interpreted to have survived through a process of natural selection in animals. Indeed, when one?s memory of an emotionally painful experience leads to serious anxiety, forgetting may produce relief. Nevertheless, an evolutionary interpretation might make it difficult to understand how the commonly gradual process of forgetting survived natural selection. In thinking about the evolution of memory together with all its possible aspects, it is helpful to consider what would happen if memories failed to fade. Forgetting clearly aids orientation in time, since old memories weaken and the new tend to stand out, providing clues for inferring duration. Without forgetting, adaptive ability would suffer; for example, learned behaviour that might have been correct a decade ago may no longer be. Cases are recorded of people who (by ordinary standards) forgot so little that their everyday activities were full of confusion. This forgetting seems to serve that survival of the individual and the species. Another line of thought assumes a memory storage system of limited capacity that provides adaptive flexibility specifically through forgetting. In this view, continual adjustments are made between learning or memory storage (input) and forgetting (output). Indeed, there is evidence that the rate at which individuals forget is directly related to how much they have learned. Such data offers gross support of contemporary models of memory that assume an input-output balance. 27. From the evolutionary point of view, __. [A] forgetting for lack of practice tends to be obviously in adaptive. [B] if a person gets very forgetful all of a sudden he must be very adaptive [C] the gradual process of forgetting is an indication of an individual?s adaptability [D] sudden forgetting may bring about adaptive consequences 28. According to the passage, if a person never forgot __. [A] he would survive best [B] he would have a lot of trouble [C] his ability to learn would be enhanced [D] the evolution of memory would stop 29. From the last paragraph we know that__. [A] forgetfulness is a response to learning [B] the memory storage system is an exactly balanced input-output system [C] memory is a compensation for forgetting [D] the capacity of a memory storage system is limited because forgetting occurs 30. In this article, the author tries to interpret the function of__. [A] remembering [B] forgetting [C] adapting [D] experiencing Part ? English-Chinese Translation The standardized educational or psychological test that are widely used to aid in selecting, classifying, assigning, or promoting students, employees, and military personnel have been the target of recent attacks in books, magazines, the daily press, and even in congress. (31) The target is wrong, for in attacking the tests, critics divert attention form the fault that lies with ill-informed or incompetent users. The tests themselves are merely tools, with characteristics that can be measured with reasonable precision under specified conditions. Whether the results will be valuable, meaningless, or even misleading depends partly upon the tool itself but largely upon the user. All informed predictions of future performance are based upon some knowledge of relevant past performance: school grades, research productivity, sales records, or whatever is appropriate. (32 )How well the predictions will be validated by later performance depends upon the amount , reliability , and appropriateness of the information used and on the skill and wisdom with which it is interpreted. Anyone who keeps careful score knows that the information available is always incomplete and that the predictions are always subject to error. Standardized tests should be considered in this context. They provide a quick, objective method of getting some kinds of information about what a person learned, the skills he has developed, or the kind of person he is. The information so obtained has, qualitatively, the same advantages and shortcomings as other kinds of information. (33) Whether to use tests. other kinds of information, or both in a particular situation depends, therefore, upon the evidence from experience concerning comparative validity and upon such factors as cost and availability. (34) In general, the tests work most effectively when the qualities to be measured can be most precisely defined and least effectively when what is to be measured or predicted cannot be well defined. Properly used, they provide a rapid means of getting comparable information about many people. Sometimes they identify students whose high potential has not been previously recognized, but there are many things they do not do. (35) For example, they do not compensate for gross social inequality, and thus do not tell how able an underprivileged youngster might have been had he grown up under more favorable circumstances. 全文翻译 广泛用来帮助选择、分类、指定或者激励学生、雇员和军队人员的标准化教育或心理测试是近来 关于书的成语关于读书的排比句社区图书漂流公约怎么写关于读书的小报汉书pdf 刊、 杂志、日报社甚至是国会攻击的目标。(31)把标准化测试作为抨击目标是错误的,因为在抨击这类测试时,批 评者未注意到其弊病在于那些对测试不甚了解或使用不当的使用者。这些测试本身只是工具,其特点能够在 特殊的条件下相当准确地被估量。测试结果是否有价值、有意义,或者甚至误导人部分取决于工具本身,而 大部分取决于使用者。 所有已知的对将来行为的预测都是基于对过去某些相关经验的了解的基础之上的:学校成绩、科研生产 力、销售业绩,或诸如此类。(32)这些预测在多大程度上被后来的表现证实,这取决于被采用信息的数量、可 靠性和适宜性以及解释这些信息的技能和才智。任何仔细记分的人都知道可用的信息总是不完整的,预测总 是容易出现错误。 标准化测试应该考虑在这样的环境中使用。他们提供快捷、客观的方法以获取关于某个人学到了什么、 他有哪些技能或者他属于哪种人的一些信息。如同其他种类的信息,如此得来的信息在质量方面也是兼有优 缺点。(33)因此,究竟是采用测试还是其他种类的信息,或是在某一特定情况下两者同时使用,须凭有关相对 效度的经验依据而定,也取决于诸如费用和现有条件等因素。 (34)一般来说,当被测定的特征能够被很准确地界定时,测试最为有效,而当被测定或被预测的东西不能 够被明确地界定时,测试的效果则最差。如果测试被恰当地使用,它们将提供一种快捷的方式来获取有关很 多人的可比较的信息。有时,通过它们,测试,还可以发现那些高潜质以前没有被发掘的学生,但许多事情 是测试做不到的。(35)例如,测试并不弥补明显的社会不公,因此,它们不能说明一个物质条件差的年轻人, 如果在较好的环境下成长,会有多大才干。 Part ? Writing (15 points) DIRECTIONS: A. Title: THE "PROJECT HOPE" B. Time limit: 40 minutes C. Word limit: 120 - 150 words (not including the given opening sentence) D. Your composition should be based on the OUTLINE below and should start with the given opening sentence: “Education plays a very important role in the modernization of our country”. E. Your composition must be written neatly on the ANSWER SHEET. OUTLINE: 1. Present situation 2. Necessity of the project 3. My suggestion 一、审题 本题为提纲式论说文。由题目可知本文讨论的是有关“希望 工程 路基工程安全技术交底工程项目施工成本控制工程量增项单年度零星工程技术标正投影法基本原理 ”的问题。 二、谋篇 按提纲所示,本文应分三方面展开。第一段应说明目前境况:一方面,贫困造成大量儿童失学,而另一方 面,国家需要大量有知识的人才。第二段,由首段的陈述自然引出实施这项工程的必要性。分析必要性切忌 写得过于空泛。这段可通过正反两方面进行论证。第三段提出自己的建议。 三、写作范文 评分标准1 15~13分:内容切题,包括提纲的全部要点,表达清楚,文字连贯,句式有变化,句子结构和用词正确。 Sample 1 (15 points) The “Hope Project” Education is the key to the modernization of our country. Chain cannot achieve its planned economic growth and catch up with those developed nations without a well-educated population. However, more than 37 million primary and secondary school students in the poverty-stricken areas of the countryside are obliged to quite school. Many of them are joining the ranks of new illiterates. Thus, a financial plan called Project Hope is carried on to rescue those children from a life of illiteracy and ignorance. The project, which brings the hope for children and promises a bright future for China, is quite necessary. As we know, being deprived of education will not only result in exhaustion of human resources in those poverty-stricken areas, but also hold back the economic development of the whole country. Developing education to arm youngsters with knowledge is an urgent task and a responsibility of the entire society. But it is impossible for the Chinese government to afford such a sum of money for hundreds of thousands of school dropouts. So a nation-wide drive, Project Hope, is the best solution. As Project Hope aims to raise funds from all walks of life, we should finance the project with money. Moreover, it needs our love and strong sense of responsibility for those children. As it says, “knowledge is a lamp leading people into the world of brightness and beauty”, so is Project Hope. 例文分析 1.提纲 ,1,现状:从国家需要人才和大量儿童失学两方面说明目前中国教育状况, ,2,需要:上学的障碍为学生与政府都没有钱, ,3,建议:捐钱及爱心等。 2(论点与谋篇 本文中心突出,结构严谨。从国家与个人两方面进行论述是本文的一大特点。它使得文章脉络非常清晰。 第二段,作者用了总——分——总的方法展开,使文章简短有力。第三段的特点是呼应主题,以俗语作总结, 并采用对照,起到了画龙点睛的作用。 3(连贯性 本文流畅不仅来自于思维的连贯,并且归功于诸多连词的应用,已在文中标出,。 4.句式及用词 经典句式:Developing education to arm youngsters with knowledge is and urgent task… As… aims to do sth.,we should… 评分标准 2 12~10分:内容切题,包括提纲的全部要点,表达比较清楚,文字基本连贯,句式有一定变化,句子结构 和用词无重大错误。 Sample 2 (10 points) The” Hope Project” Education plays a very important role in the modernization of our country. In rural areas millions of children have no chance to go to school. Helping their family in the field or doing endless housework at home, they do not even know how to write their own names. These new illiterates are impossible to catch the change of the world. So the Project Hope was established to help the poor children. The Project is quite necessary. On the one hand, these children are too poor to buy the books and go to school. On the other hand, as a developing country China is not likely to spend a huge amount of money on education. Only through the love the project can provide more and more education chances for the poor children. Thus, everyone should do what he can for the Project Hope. I think this project is great important to our country. So my suggestion is that people should donate our money and the government should encourage more of the public to participate the project. And I?m sure the project will be fruitful. 例文分析 1(提纲 ,1,儿童失学,提出希望工程, ,2,从两方面说明工程的必要性, ,3,提出建议。 2(论点与谋篇 本文论点突出,按照提纲所示将文章按三段展开。第一段的缺点在于:第一句与第二句间缺乏过渡。第二 句应展开说明教育对中国的重要性,然后再进行转折衔接第三句,这样内容较衔接。但此段的描写较为生动。 不是直接讲述,而是通过形象地描述,用干活、做家务及不会写名字的对比生动展示失学儿童的现状。 3(连贯性 段与段之间衔接比较自然。 4.句式及用词 典型错误: 倒装:Only through the love the project can provide more and more education chances for the poor children. Only开头的句子应为部分倒装。应为:Only through the love can the project provide more and more education chances for the poor children. 1996年年全真试题 Part ?Cloze Test Directions: For each numbered blank in the following passage, there are four choices marked ,A,, ,B,, ,C, and [D]. Choose the best one and mark your answer on ANSWER SHEET 1 by blackening the corresponding letter in the brackets. (10 points) Vitamins are organic compounds necessary in small amounts in the diet for the normal growth and maintenance of life of animals, including man. They do not provide energy, 1 do they construct or build any part of the body. They are needed for 2 foods into energy and body maintenance. There are thirteen or more of them, and if 3 is missing a deficiency disease becomes 4 . Vitamins are similar because they are made of the same elements—usually carbon, hydrogen, oxygen, and 5 nitrogen. They are different 6 their elements are arranged differently, and each vitamin 7 one or more specific functions in the body. 8 enough vitamins is essential to life, although the body has no nutritional use for 9 vitamins. Many people, 10 , believe in being on the “safe side” and thus take extra vitamins. However, a well-balanced diet will usually meet all the body?s vitamin needs. 1.,A,either ,B,so ,C,nor ,D,never 2.,A,shifting ,B,transferring ,C,altering ,D,transforming 3.,A,any ,B,some ,C,anything ,D,something 4.,A,serious ,B,apparent ,C,severe ,D,fatal 5.,A,mostly ,B,partially ,C,sometimes ,D,rarely 6.,A,in that ,B,so that ,C,such that ,D,except that 7.,A,undertakes ,B,holds ,C,plays ,D,performs 8.,A,Supplying ,B,Getting ,C,Providing ,D,Furnishing 9.,A,exceptional ,B,exceeding ,C,excess ,D,external 10.,A,nevertheless ,B,therefore ,C,moreover ,D,meanwhile Part ?Reading Comprehension Directions: Each of the passages below is followed by some questions. For each questions there are four answers marked,A,, ,B,,,C,and,D,. Read the passages carefully and choose the best answer to each of the questions. Then mark your answer on ANSWER SHEET 1 by blackening the corresponding letter in the brackets with a pencil. (40 points) Passage 1 Tight-lipped elders used to say, “It?s not what you want in this world, but what you get.” Psychology teaches that you do get what you want if you know what you want and want the right things. You can make a mental blueprint of a desire as you would make a blueprint of a house, and each of us is continually making these blueprints in the general routine of everyday living. If we intend to have friends to dinner, we plan the menu, make a shopping list, and decide which food to cook first, and such planning is an essential for any type of meal to be served. Likewise, if you want to find a job, take a sheet of paper, and write a brief account of yourself. In making a blueprint for a job, begin with yourself, for when you know exactly what you have to offer, you can intelligently plan where to sell your services. This account of yourself is actually a sketch of your working life and should include education, experience and references. Such an account is valuable. It can be referred to in filling out standard application blanks and is extremely helpful in personal interviews. While talking to you, your could be employer is deciding whether your education, your experience, and other qualifications will pay him to employ you and your “wares” and abilities must be displayed in an orderly and reasonably connected manner. When you have carefully prepared a blueprint of your abilities and desires, you have something tangible to sell. Then you are ready to hunt for a job. Get all the possible information about your could be job. Make inquiries as to the details regarding the job and the firm. Keep your eyes and ears open, and use your own judgment. Spend a certain amount of time each day seeking the employment you wish for, and keep in mind: Securing a job is your job now. 11. What do the elders mean when they say, “It?s not what you want in this world, but what you get.”? ,A,You?ll certainly get what you want. ,B,It?s no use dreaming. ,C,You should be dissatisfied with what you have. ,D,It?s essential to set a goal for yourself. 12. A blueprint made before inviting a friend to dinner is used in this passage as . ,A,an illustration of how to write an application for a job ,B,an indication of how to secure a good job ,C,a guideline for job description ,D,a principle for job evaluation 13. According to the passage, one must write an account of himself before starting to find a job because . ,A,that is the first step to please the employer ,B,that is the requirement of the employer ,C,it enables him to know when to sell his services ,D,it forces him to become clearly aware of himself 14. When you have carefully prepared a blueprint of your abilities and desires, you have something . ,A,definite to offer ,B,imaginary to provide ,C,practical to supply ,D,desirable to present Passage 2 With the start of BBC World Service Television, millions of viewers in Asia and America can now watch the Corporation?s news coverage, as well as listen to it. And of course in Britain listeners and viewers can tune in to two BBC television channels, five BBC national radio services and dozens of local radio stations. They are brought sport, comedy, drama, music, news and current affairs, education, religion, parliamentary coverage, children?s programmes and films for an annual licence fee of ,83 per household. It is a remarkable record, stretching back over 70 years — yet the BBC?s future is now in doubt. The Corporation will survive as a publicly funded broadcasting organization, at least for the time being, but its role, its size and its programmes are now the subject of a nationwide debate in Britain. The debate was launched by the Government, which invited anyone with an opinion of the BBC — including ordinary listeners and viewers — to say what was good or bad about the Corporation, and even whether they thought it was worth keeping. The reason for its inquiry is that the BBC?s royal charter runs out in 1996 and it must decide whether to keep the organization as it is, or to make changes. Defenders of the Corporation — of whom there are many — are fond of quoting the American slogan “If it isn?t broke, don?t fix it.” The BBC “ain?t broke”, they say, by which they mean it is not broken (as distinct from the word „broke?, meaning having no money), so why bother to change it? Yet the BBC will have to change, because the broadcasting world around it is changing. The commercial TV channels —— ITV and Channel 4 —— were required by the Thatcher Government?s Broadcasting Act to become more commercial, competing with each other for advertisers, and cutting costs and jobs. But it is the arrival of new satellite channels — funded partly by advertising and partly by viewers? subscriptions — which will bring about the biggest changes in the long term. 15. The world famous BBC now faces . ,A,the problem of news coverage ,B,an uncertain prospect ,C,inquiries by the general public ,D,shrinkage of audience 16. In the passage, which of the following about the BBC is not mentioned as the key issue? ,A,Extension of its TV service to Far East. ,B,Programmes as the subject of a nation-wide debate. ,C,Potentials for further international co-operations. ,D,Its existence as a broadcasting organization. 17. The BBC?s “royal charter” (Line 4, Paragraph 4) stands for . ,A,the financial support from the royal family. ,B,the privileges granted by the Queen. ,C,a contract with the Queen. ,D,a unique relationship with the royal family. 18. The foremost reason why the BBC has to readjust itself is no other than . ,A,the emergence of commercial TV channels.,B,the enforcement of Broadcasting Act by the government. ,C,the urgent necessity to reduce costs and jobs. ,D,the challenge of new satellite channels. Passage 3 In the last half of the nineteenth century “capital” and “labour” were enlarging and perfecting their rival organizations on modern lines. Many an old firm was replaced by a limited liability company with a bureaucracy of salaried managers. The change met the technical requirements of the new age by engaging a large professional element and prevented the decline in efficiency that so commonly spoiled the fortunes of family firms in the second and third generation after the energetic founders. It was moreover a step away from individual initiative, towards collectivism and municipal and state-owned business. The railway companies, though still private business managed for the benefit of shareholders, were very unlike old family business. At the same time the great municipalities went into business to supply lighting, trams and other services to the taxpayers. The growth of the limited liability company and municipal business had important consequences. Such large, impersonal manipulation of capital and industry greatly increased the numbers and importance of shareholders as a class, an element in national life representing irresponsible wealth detached from the land and the duties of the landowners; and almost equally detached from the responsible management of business. All through the nineteenth century, America, Africa, India, Australia and parts of Europe were being developed by British capital, and British shareholders were thus enriched by the world?s movement towards industrialization. Towns like Bournemouth and Eastbourne sprang up to house large “comfortable” classes who had retired on their incomes, and who had no relation to the rest of the community except that of drawing dividends and occasionally attending a shareholders? meeting to dictate their orders to the management. On the other hand “shareholding” meant leisure and freedom which was used by many of the later Victorians for the highest purpose of a great civilization. The “shareholders” as such had no knowledge of the lives, thoughts or needs of the workmen employed by the company in which he held shares, and his influence on the relations of capital and labor was not good. The paid manager acting for the company was in more direct relation with the men and their demands, but even he had seldom that familiar personal knowledge of the workmen which the employer had often had under the more patriarchal system of the old family business now passing away. Indeed the mere size of operations and the numbers of workmen involved rendered such personal relations impossible. Fortunately, however, the increasing power and organization of the trade unions, at least in all skilled trades, enabled the workmen to meet on equal terms the managers of the companies who employed them. The cruel discipline of the strike and lockout taught the two parties to respect each other?s strength and understand the value of fair negotiation. 19. It?s true of the old family firms that . ,A,they were spoiled by the younger generations ,B,they failed for lack of individual initiative ,C,they lacked efficiency compared with modern companies ,D,they could supply adequate services to the taxpayers 20. The growth of limited liability companies resulted in . ,A,the separation of capital from management ,B,the ownership of capital by managers ,C,the emergence of capital and labour as two classes ,D,the participation of shareholders in municipal business 21. According to the passage, all of the following are true except that . ,A,the shareholders were unaware of the needs of the workers ,B,the old firm owners had a better understanding of their workers ,C,the limited liability companies were too large to run smoothly ,D,the trade unions seemed to play a positive role 22. The author is most critical of . ,A,family firm owners ,B,landowners ,C,managers ,D,shareholders Passage 4 What accounts for the great outburst of major inventions in early America— breakthroughs such as the telegraph, the steamboat and the weaving machine? Among the many shaping factors, I would single out the country?s excellent elementary schools; a labor force that welcomed the new technology; the practice of giving premiums to inventors; and above all the American genius for nonverbal, “spatial” thinking about things technological. Why mention the elementary schools? Because thanks to these schools our early mechanics, especially in the New England and Middle Atlantic states, were generally literate and at home in arithmetic and in some aspects of geometry and trigonometry. Acute foreign observers related American adaptiveness and inventiveness to this educational advantage. As a member of a British commission visiting here in 1853 reported, “With a mind prepared by thorough school discipline, the American boy develops rapidly into the skilled workman.” A further stimulus to invention came from the “premium” system, which preceded our patent system and for years ran parallel with it. This approach, originated abroad, offered inventors medals, cash prizes and other incentives. In the United States, multitudes of premiums for new devices were awarded at country fairs and at the industrial fairs in major cities. Americans flocked to these fairs to admire the new machines and thus to renew their faith in the beneficence of technological advance. Given this optimistic approach to technological innovation, the American worker took readily to that special kind of nonverbal thinking required in mechanical technology. As Eugene Ferguson has pointed out, “A technologist thinks about objects that cannot be reduced to unambiguous verbal descriptions; they are dealt with in his mind by a visual, nonverbal process … The designer and the inventor … are able to assemble and manipulate in their minds devices that as yet do not exist.” This nonverbal “spatial” thinking can be just as creative as painting and writing. Robert Fulton once wrote, “The mechanic should sit down among levers, screws, wedges, wheels, etc, like a poet among the letters of the alphabet, considering them as an exhibition of his thoughts, in which a new arrangement transmits a new idea.” When all these shaping forces—schools, open attitudes, the premium system, a genius for spatial thinking —interacted with one another on the rich U.S. mainland, they produced that American characteristic emulation. Today that word implies mere imitation. But in earlier times it meant a friendly but competitive striving for fame and excellence. 23. According to the author, the great outburst of major inventions in early America was in a large part due to . ,A,elementary schools ,B,enthusiastic workers ,C,the attractive premium system ,D,a special way of thinking 24. It is implied that adaptiveness and inventiveness of the early American mechanics . ,A,benefited a lot from their mathematical knowledge. ,B,shed light on disciplined school management. ,C,was brought about by privileged home training. ,D,owed a lot to the technological development. 25. A technologist can be compared to an artist because . ,A,they are both winners of awards. ,B,they are both experts in spatial thinking. ,C,they both abandon verbal description ,D,they both use various instruments 26. The best title for this passage might be . ,A,Inventive Mind ,B,Effective Schooling ,C,Ways of Thinking ,D,Outpouring of Inventions Passage 5 Rumor has it that more than 20 books on creationism/evolution are in the publisher?s pipelines. A few have already appeared. The goal of all will be to try to explain to a confused and often unenlightened citizenry that there are not two equally valid scientific theories for the origin and evolution of universe and life. Cosmology, geology, and biology have provided a consistent, unified, and constantly improving account of what happened. “Scientific” creationism, which is being pushed by some for “equal time” in the classrooms whenever the scientific accounts of evolution are given, is based on religion, not science. Virtually all scientists and the majority of nonfundamentalist religious leaders have come to regard “scientific” creationism as bad science and bad religion. The first four chapters of Kitcher?s book give a very brief introduction to evolution. At appropriate places, he introduces the criticisms of the creationists and provides answers. In the last three chapters, he takes off his gloves and gives the creationists a good beating. He describes their programmes and tactics, and, for those unfamiliar with the ways of creationists, the extent of their deception and distortion may come as an unpleasant surprise. When their basic motivation is religious, one might have expected more Christian behavior. Kitcher is a philosopher, and this may account, in part, for the clarity and effectiveness of his arguments. The non-specialist will be able to obtain at least a notion of the sorts of data and argument that support evolutionary theory. The final chapters on the creationists will be extremely clear to all. On the dust jacket of this fine book, Stephen Jay Gould says: “This book stands for reason itself.” And so it does - and all would be well were reason the only judge in the creationism/evolution debate. 27. “Creationism” in the passage refers to . ,A,evolution in its true sense as to the origin of the universe ,B,a notion of the creation of religion ,C,the scientific explanation of the earth formation ,D,the deceptive theory about the origin of the universe 28. Kitcher?s book is intended to . ,A,recommend the views of the evolutionists ,B,expose the true features of creationists ,C,curse bitterly at his opponents ,D,launch a surprise attack on creationists 29. From the passage we can infer that . ,A,reasoning has played a decisive role in the debate ,B,creationists do not base their argument on reasoning ,C,evolutionary theory is too difficult for non-specialists ,D,creationism is supported by scientific findings 30. This passage appears to be a digest of . ,A,a book review ,B,a scientific paper ,C,a magazine feature ,D,a newspaper editorial Part ? English—Chinese Translation Directions: Read the following text carefully and then translate the underlined segments into Chinese. Your translation should be written clearly on ANSWER SHEET 2. (15 points) The differences in relative growth of various areas of scientific research have several causes. 31)Some of these causes are completely reasonable results of social needs. Others are reasonable consequences of particular advances in science being to some extent self-accelerating. Some, however, are less reasonable processes of different growth in which preconceptions of the form scientific theory ought to take, by persons in authority, act to alter the growth pattern of different areas. This is a new problem probably not yet unavoidable; but it is a frightening trend. 32)This trend began during the Second World War, when several governments came to the conclusion that the specific demands that a government wants to make of its scientific establishment cannot generally be foreseen in detail. It can be predicted, however, that from time to time questions will arise which will require specific scientific answers. It is therefore generally valuable to treat the scientific establishment as a resource or machine to be kept in functional order. 33)This seems mostly effectively done by supporting a certain amount of research not related to immediate goals but of possible consequence in the future. This kind of support, like all government support, requires decisions about the appropriate recipients of funds. Decisions based on utility as opposed to lack of utility are straightforward. But a decision among projects none of which has immediate utility is more difficult. The goal of the supporting agencies is the praisable one of supporting “good” as opposed to “bad” science, but a valid determination is difficult to make. Generally, the idea of good science tends to become confused with the capacity of the field in question to generate an elegant theory. 34)However, the world is so made that elegant systems are in principle unable to deal with some of the world?s more fascinating and delightful aspects. 35)New forms of thought as well as new subjects for thought must arise in the future as they have in the past, giving rise to new standards of elegance. Section ? Writing Directions: A. Title: GOOD HEALTH B. Time limit: 40minutes C. Word limit: 120—150 words (not including the given opening sentence) D. Your composition should be based on the “OUTLINE” below and should start with the given opening sentence: “The desire for good health is universal”. E. Your composition must be written clearly on the ANSWER SHEET. Outline: 1. Importance of good health. 2. Ways to keep fit. 3. My own practices. Section ? Writing 一、审题与谋篇 本文命题形式为提纲式控制性写作,提纲有三点要求:健康的重要性,保持健康身体的方法,我自己的 实践。并给出了起始句The desire for good health is universal。 文章内容以健康为中心,而且从提纲也可看出,本文首段重在议论,后两段重在说明,整体为议论和说 明相结合。根据提纲所示,本文分三段展开。第一段,突出健康的重要性。第二段,说明保持身体健康的途 径,可用列举的方法进行说明,必要时稍加展开。第三段,描述自己的切身经历。但应注意,第三段的自身 做法应呼应第二段的举例,这样文章结构会比较严谨。 二、参考范文 GOOD HEALTH The desire for good health is universal. Wherever you are and whatever you do,staying healthy precedes a successful career and a happy life. People with good health can do work with full energy and confidence and their progress in turn contributes to their health and happiness. On the contrary, a sick one usually lacks the vigor and interest to fulfill his or her role in life, which deprives him or her of many opportunities to become successful and happy. Realizing the importance of good health is far from being enough. We must do something effective to keep fit. The best place to begin is at the dinner table. Eating less junk food and having a balanced diet is the first step for most people. The next step is to exercise regularly. Vigorous exercise can benefit not only the muscles but also the organs. Last but not the least, don?t damage the body with drugs,including cigarettes and too much alcohol. As far as I am is concerned,I am neither indulged in food nor in such harmful substances as cigarettes or alcohol. Moreover, doing exercises is part of my routine life. Therefore, I am in good shape and always energetic. 三、范文点评 文章结构: 从结构上看,该范文按照提纲要求,分为三段论述。第一段采用了先总后分的写作方法,段首为题目中 给出的主题句,接下来用正反对比的写法进行论证,使人印象深刻。第二段段首也是主题句,并用The best place to begin、The next step、和Last but not the least分别引导对保持身体健康提出的三点建议,段落内部层次分明。 第三段采用先分后总的写作方法,呼应上段的建议,谈了自己的实际情况,并在段尾进行了总结。 语言亮点: 1. universal:普遍的。例如:It is a universally accepted idea that …,…是一个普遍接受的观点,。 2. precede:在…之前。可代替的表达有:is the basis of或is the foundation of。 3. in turn:反过来。类似用法的词还有consequently,因而,。 4. on the contrary:“与此相反”,类似表示对照的短语有:unlike, in contrast, whereas, rather than, conversely, instead, by contrast等。 5. deprive sb. of sth.:剥夺某人某物,或某种权利,。例如:Many children are deprived of their rights to receive education because of poverty.,很多孩子由于贫穷而被剥夺了受教育的权利,。 6. is far from:原不…,远非…。例如:The present situation is far from being satisfying.,目前的情况远非 令人满意,。 7. junk food:“垃圾食品”,指一些没有营养的食物,。考生注意收集有关食物的词汇,如:take out,外卖,, fast food,快餐, 8. a balanced diet:一个均衡的饮食。 9. Last but not the least:最后一点,但并不是最不重要的一点。 10. As far as I am is concerned:“至于我自己”,as far as … be concerned谈到…,至于…。用于提起话题。 11. indulge:be indulged in …沉湎于…。 12. routine life:日常生活。routine为regular的近义词。 四、写作误区 篇章结构误区: 考生需要避免的第一个写作误区是跑题。本题在第三段中,跑题现象较为明显,本段要求考生谈谈自己 的做法,但有的考生对practices产生误解,把它当作“实践,与理论相对应,”,而写成“实践是很重要的”或“实 践是检验真理的唯一标准”,结果跑题了。还有的考生在谋篇上不够周全,将第一段写得过于庞大,而末段又 过于简短,造成文章整体比例失调,头重脚轻,因此丢分。也有的同学将第二段写得过于详尽,举例过细, 重在描写刻画,而非说明,也是误解本题初衷的表现。 语言表达错误: ?词义冗余: In my opinion, I think a good diet is the most important thing. (In my opinion / I think a good diet is the most important thing.) ?词义错误: Practice is very important to health. (Exercise is very important to health.) ?句子结构混乱: So careful the health, not just think medicine can care for all the illness.,So pay attention to your health, and do not just think medicine can cure all the illness., ?累赘: Eating no food shouldn?t be considered as the only way to solve the problem of keeping health.,Eating little is not the only way to keep health., ?句子结构不平行: We should try our best to keep healthy by taking exercises and don?t eat too much or too little.,We should try our best to keep healthy by taking exercises and having a balanced diet., ?不间断句子: People?s living standard improved, more and more people began to worry about their health. ,As people?s living standard improved, more and more people began to care about their health., ?残句: I think, to have both physical and mental health to succeed in the competitive society.,Both physical and mental health are the key to success in the competitive society., 1997年全真试题 Part ?Cloze Test Directions: For each numbered blank in the following passage, there are four choices marked ,A,, ,B,, ,C, and ,D,. Choose the best one and mark your answer on ANSWER SHEET 1 by blackening the corresponding letter in the brackets. (10 points) Manpower Inc., with 560 000 workers, is the world?s largest temporary employment agency. Every morning, its people 1 into the offices and factories of America, seeking a day?s work for a day?s pay. One day at a time. 2 industrial giants like General Motors and IBM struggle to survive 3 reducing the number of employees, Manpower, based in Milwaukee, Wisconsin, is booming. 4 its economy continues to recover, the US is increasingly becoming a nation of part- timers and temporary workers. This “ 5 ” work force is the most important 6 in American business today, and it is 7 changing the relationship between people and their jobs. The phenomenon provides a way for companies to remain globally competitive 8 avoiding market cycles and the growing burdens 9 by employment rules, health care costs and pension plans. For workers it can mean an end to the security, benefits and sense of 10 that came from being a loyal employee. 1.,A,swarm ,B,stride ,C,separate ,D,slip 2.,A,For ,B,Because ,C,As ,D,Since 3.,A,from ,B,in ,C,on ,D,by 4.,A,Even though ,B,Now that ,C,If only ,D,Provided that 5.,A,durable ,B,disposable ,C,available ,D,transferable 6.,A,approach ,B,flow ,C,fashion ,D, trend 7.,A,instantly ,B,reversely ,C,fundamentally ,D,sufficiently 8.,A,but ,B,while ,C,and ,D,whereas 9.,A,imposed ,B,restricted ,C,illustrated ,D,confined 10.,A,excitement ,B,conviction ,C,enthusiasm ,D,importance Part ?Reading Comprehension Directions: Each of the passages below is followed by some questions. For each question there are four answers marked,A,, ,B,,,C,and,D,. Read the passages carefully and choose the best answer to each of the questions. Then mark your answer on ANSWER SHEET 1 by blackening the corresponding letter in the brackets. (40 points) Passage 1 It was 3: 45 in the morning when the vote was finally taken. After six months of arguing and final 16 hours of hot parliamentary debates, Australia?s Northern Territory became the first legal authority in the world to allow doctors to take the lives of incurably ill patients who wish to die. The measure passed by the convincing vote of 15 to 10. Almost immediately word flashed on the Internet and was picked up, half a world away, by John Hofsess, executive director of the Right to Die Society of Canada. He sent it on via the group?s on-line service, Death NET. Says Hofsess: “We posted bulletins all day long, because of course this isn?t just something that happened in Australia. It?s world history.” The full import may take a while to sink in. The NT Rights of the Terminally Ill law has left physicians and citizens alike trying to deal with its moral and practical implications. Some have breathed sighs of relief, others, including churches, right-to-life groups and the Australian Medical Association, bitterly attacked the bill and the haste of its passage. But the tide is unlikely to turn back. In Australia—where an aging population, life-extending technology and changing community attitudes have all played their part—other states are going to consider making a similar law to deal with euthanasia. In the US and Canada, where the right-to-die movement is gathering strength, observers are waiting for the dominoes to start falling. Under the new Northern Territory law, an adult patient can request death—probably by a deadly injection or pill—to put an end to suffering. The patient must be diagnosed as terminally ill by two doctors. After a “cooling off” period of seven days, the patient can sign a certificate of request. After 48 hours the wish for death can be met. For Lloyd Nickson, a 54-year-old Darwin resident suffering from lung cancer, the NT Rights of Terminally Ill law means he can get on with living without the haunting fear of his suffering: a terrifying death from his breathing condition. “I?m not afraid of dying from a spiritual point of view, but what I was afraid of was how I?d go, because I?ve watched people die in the hospital fighting for oxygen and clawing at their masks, ” he says. 11. From the second paragraph we learn that . ,A,the objection to euthanasia is slow to come in other countries ,B,physicians and citizens share the same view on euthanasia ,C,changing technology is chiefly responsible for the hasty passage of the law ,D,it takes time to realize the significance of the law?s passage 12. When the author says that observers are waiting for the dominoes to start falling, he means . ,A,observers are taking a wait-and-see attitude towards the future of euthanasia ,B,similar bills are likely to be passed in the US, Canada and other countries ,C,observers are waiting to see the result of the game of dominoes ,D,the effect-taking process of the passed bill may finally come to a stop 13. When Lloyd Nickson dies, he will . ,A,face his death with calm characteristic of euthanasia ,B,experience the suffering of a lung cancer patient ,C,have an intense fear of terrible suffering ,D,undergo a cooling off period of seven days 14. The author?s attitude towards euthanasia seems to be that of . ,A,opposition ,B,suspicion ,C,approval ,D,indifference Passage 2 A report consistently brought back by visitors to the US is how friendly, courteous, and helpful most Americans were to them. To be fair, this observation is also frequently made of Canada and Canadians, and should best be considered North American. There are, of course, exceptions. Small-minded officials, rude waiters, and ill-mannered taxi drivers are hardly unknown in the US. Yet it is an observation made so frequently that it deserves comment. For a long period of time and in many parts of the country, a traveler was a welcome break in an otherwise dull existence. Dullness and loneliness were common problems of the families who generally lived distant from one another. Strangers and travelers were welcome sources of diversion, and brought news of the outside world. The harsh realities of the frontier also shaped this tradition of hospitality. Someone traveling alone, if hungry, injured, or ill, often had nowhere to turn except to the nearest cabin or settlement. It was not a matter of choice for the traveler or merely a charitable impulse on the part of the settlers. It reflected the harshness of daily life: if you didn?t take in the stranger and take care of him, there was no one else who would. And someday, remember, you might be in the same situation. Today there are many charitable organizations which specialize in helping the weary traveler. Yet, the old tradition of hospitality to strangers is still very strong in the US, especially in the smaller cities and towns away from the busy tourist trails. “I was just traveling through, got talking with this American, and pretty soon he invited me home for dinner—amazing.” Such observations reported by visitors to the US are not uncommon, but are not always understood properly. The casual friendliness of many Americans should be interpreted neither as superficial nor as artificial, but as the result of a historically developed cultural tradition. As is true of any developed society, in America a complex set of cultural signals, assumptions, and conventions underlies all social interrelationships. And, of course, speaking a language does not necessarily mean that someone understands social and cultural patterns. Visitors who fail to “translate” cultural meanings properly often draw wrong conclusions. For example, when an American uses the word “friend”, the cultural implications of the word may be quite different from those it has in the visitor?s language and culture. It takes more than a brief encounter on a bus to distinguish between courteous convention and individual interest. Yet, being friendly is a virtue that many Americans value highly and expect from both neighbors and strangers. 15. In the eyes of visitors from the outside world ,. ,A,rude taxi drivers are rarely seen in the US ,B,small-minded officials deserve a serious comment ,C,Canadians are not so friendly as their neighbors ,D,most Americans are ready to offer help 16. It could be inferred from the last paragraph that . ,A,culture exercises an influence over social interrelationship ,B,courteous convention and individual interest are interrelated ,C,various virtues manifest themselves exclusively among friends ,D,social interrelationships equal the complex set of cultural conventions 17. Families in frontier settlements used to entertain strangers . ,A,to improve their hard life ,B,in view of their long-distance travel ,C,to add some flavor to their own daily life ,D,out of a charitable impulse 18. The tradition of hospitality to strangers . ,A,tends to be superficial and artificial ,B,is generally well kept up in the United States ,C,is always understood properly ,D,has something to do with the busy tourist trails Passage 3 Technically, any substance other than food that alters our bodily or mental functioning is a drug. Many people mistakenly believe the term drug refers only to some sort of medicine or an illegal chemical taken by drug addicts. They don?t realize that familiar substances such as alcohol and tobacco are also drugs. This is why the more neutral term substance is now used by many physicians and psychologists. The phrase “substance abuse” is often used instead of “drug abuse” to make clear that substances such as alcohol and tobacco can be just as harmfully misused as heroin and cocaine. We live in a society in which the medical and social use of substances (drugs) is pervasive: an aspirin to quiet a headache, some wine to be sociable, coffee to get going in the morning, a cigarette for the nerves. When do these socially acceptable and apparently constructive uses of a substance become misuses? First of all, most substances taken in excess will produce negative effects such as poisoning or intense perceptual distortions. Repeated use of a substance can also lead to physical addiction or substance dependence. Dependence is marked first by an increased tolerance, with more and more of the substance required to produce the desired effect, and then by the appearance of unpleasant withdrawal symptoms when the substance is discontinued. Drugs (substances) that affect the central nervous system and alter perception, mood, and behavior are known as psychoactive substances. Psychoactive substances are commonly grouped according to whether they are stimulants, depressants, or hallucinogens. Stimulants initially speed up or activate the central nervous system, whereas depressants slow it down. Hallucinogens have their primary effect on perception, distorting and altering it in a variety of ways including producing hallucinations. These are the substances often called psychedelic (from the Greek word meaning “mind-manifestation”) because they seemed to radically alter one?s state of consciousness. 19. “Substance abuse”(Line 5, Paragraph 1) is preferable to “drug abuse” in that . ,A,substances can alter our bodily or mental functioning if illegally used ,B,“drug abuse” is only related to a limited number of drugtakers ,C,alcohol and tobacco are as fatal as heroin and cocaine ,D,many substances other than heroin or cocaine can also be poisonous 20. The word “pervasive” (Line 1, Paragraph 2) might mean . ,A,widespread ,B,overwhelming ,C,piercing ,D,fashionable 21. Physical dependence on certain substances results from . ,A,uncontrolled consumption of them over long periods of time ,B,exclusive use of them for social purposes ,C,quantitative application of them to the treatment of diseases ,D,careless employment of them for unpleasant symptoms 22. From the last paragraph we can infer that . ,A,stimulants function positively on the mind ,B,hallucinogens are in themselves harmful to health ,C,depressants are the worst type of psychoactive substances ,D,the three types of psychoactive substances were commonly used in groups Passage 4 No company likes to be told it is contributing to the moral decline of a nation. “Is this what you intended to accomplish with your careers?” Senator Robert Dole asked Time Warner executives last week. “You have sold your souls, but must you corrupt our nation and threaten our children as well?” At Time Warner, however, such questions are simply the latest manifestation of the soul-searching that has involved the company ever since the company was born in 1990. It?s a self-examination that has, at various times, involved issues of responsibility, creative freedom and the corporate bottom line. At the core of this debate is chairman Gerald Levin, 56, who took over for the late Steve Ross in 1992. On the financial front, Levin is under pressure to raise the stock price and reduce the company?s mountainous debt, which will increase to $ 17.3 billion after two new cable deals close. He has promised to sell off some of the property and restructure the company, but investors are waiting impatiently. The flap over rap is not making life any easier for him. Levin has consistently defended the company?s rap music on the grounds of expression. In 1992, when Time Warner was under fire for releasing Ice-T?s violent rap song Cop Killer, Levin described rap as a lawful expression of street culture, which deserves an outlet. “The test of any democratic society, ”he wrote in a Wall Street Journal column, “lies not in how well it can control expression but in whether it gives freedom of thought and expression the widest possible latitude, however disputable or irritating the results may sometimes be. We won?t retreat in the face of any threats.” Levin would not comment on the debate last week, but there were signs that the chairman was backing off his hard-line stand, at least to some extent. During the discussion of rock singing verses at last month?s stockholders? meeting, Levin asserted that “music is not the cause of society?s ills” and even cited his son, a teacher in the Bronx, New York, who uses rap to communicate with students. But he talked as well about the “balanced struggle” between creative freedom and social responsibility, and he announced that the company would launch a drive to develop standards for distribution and labeling of potentially objectionable music. The 15-member Time Warner board is generally supportive of Levin and his corporate strategy. But insiders say several of them have shown their concerns in this matter. “Some of us have known for many, many years that the freedoms under the First Amendment are not totally unlimited, ” says Luce. “I think it is perhaps the case that some people associated with the company have only recently come to realize this.” 23. Senator Robert Dole criticized Time Warner for . ,A,its raising of the corporate stock price ,B,its self-examination of soul ,C,its neglect of social responsibility ,D,its emphasis on creative freedom 24. According to the passage, which of the following is TRUE? ,A,Luce is a spokesman of Time Warner. ,B,Gerald Levin is liable to compromise. ,C,Time Warner is united as one in the face of the debate. ,D,Steve Ross is no longer alive 25. In face of the recent attacks on the company, the chairman . ,A,stuck to a strong stand to defend freedom of expression,B,softened his tone and adopted some new policy ,C,changed his attitude and yielded to objection ,D,received more support from the 15-member board 26. The best title for this passage might be . ,A,A Company under Fire ,B,A Debate on Moral Decline ,C,A Lawful Outlet of Street Culture ,D,A Form of Creative Freedom Passage 5 Much of the language used to describe monetary policy, such as “steering the economy to a soft landing” or “a touch on the brakes”, makes it sound like a precise science. Nothing could be further from the truth. The link between interest rates and inflation is uncertain. And there are long, variable lags before policy changes have any effect on the economy. Hence the analogy that likens the conduct of monetary policy to driving a car with a blackened windscreen, a cracked rear-view mirror and a faulty steering wheel. Given all these disadvantages, central bankers seem to have had much to boast about of late. Average inflation in the big seven industrial economies fell to a mere 2.3% last year, close to its lowest level in 30 years, before rising slightly to 2.5% this July. This is a long way below the double-digit rates which many countries experienced in the 1970s and early 1980s. It is also less than most forecasters had predicted. In late 1994 the panel of economists which The Economist polls each month said that America?s inflation rate would average 3.5% in 1995. In fact, it fell to 2.6% in August, and is expected to average only about 3% for the year as a whole. In Britain and Japan inflation is running half a percentage point below the rate predicted at the end of last year. This is no flash in the pan; over the past couple of years, inflation has been consistently lower than expected in Britain and America. Economists have been particularly surprised by favourable inflation figures in Britain and the United States, since conventional measures suggest that both economies, and especially America?s, have little productive slack. America?s capacity utilisation, for example, hit historically high levels earlier this year, and its jobless rate (5.6% in August) has fallen below most estimates of the natural rate of unemployment—the rate below which inflation has taken off in the past. Why has inflation proved so mild? The most thrilling explanation is, unfortunately, a little defective. Some economists argue that powerful structural changes in the world have up-ended the old economic models that were based upon the historical link between growth and inflation. 27. From the passage we learn that . ,A,there is a definite relationship between inflation and interest rates ,B,economy will always follow certain models ,C,the economic situation is better than expected ,D,economists had foreseen the present economic situation 28. According to the passage, which of the following is TRUE? ,A,Making monetary policies is comparable to driving a car. ,B,An extremely low jobless rate will lead to inflation. ,C,A high unemployment rate will result from inflation. ,D,Interest rates have an immediate effect on the economy. 29. The sentence “This is no flash in the pan” (Line 5, Paragraph 3) means that . ,A,the low inflation rate will last for some time ,B,the inflation rate will soon rise ,C,the inflation will disappear quickly ,D,there is no inflation at present 30. The passage shows that the author isthe present situation . ,A,critical of ,B,puzzled by ,C,disappointed at ,D,amazed at Part ? English-Chinese Translation Directions: Read the following passage carefully and then translate the underlined segments into Chinese. Your translation must be written clearly on ANSWER SHEET 2. (15 points) Do animals have rights? This is how the question is usually put. It sounds like a useful, ground-clearing way to start. 31)Actually, it isn?t, because it assumes that there is an agreed account of human rights, which is something the world does not have. On one view of rights, to be sure, it necessarily follows that animals have none.32)Some philosophers argue that rights exist only within a social contract, as part of an exchange of duties and entitlements. Therefore animals cannot have rights. The idea of punishing a tiger that kills somebody is absurd, for exactly the same reason, so is the idea that tigers have rights. However, this is only one account, and by no means an uncontested one. It denies rights not only to animals but also to some people—for instance, to infants, the mentally incapable and future generations. In addition, it is unclear what force a contract can have for people who never consented to it: how do you reply to somebody who says “ I don?t like this contract”? The point is this: without agreement on the rights of people, arguing about the rights of animals is fruitless.33)It leads the discussion to extremes at the outset: it invites you to think that animals should be treated either with the consideration humans extend to other humans, or with no consideration at all. This is a false choice. Better to start with another, more fundamental, question: is the way we treat animals a moral issue at all? Many deny it.34)Arguing from the view that humans are different from animals in every relevant respect, extremists of this kind think that animals lie outside the area of moral choice. Any regard for the suffering of animals is seen as a mistake—a sentimental displacement of feeling that should properly be directed to other humans. This view, which holds that torturing a monkey is morally equivalent to chopping wood, may seem bravely “logical”. In fact it is simply shallow: the confused centre is right to reject it. The most elementary form of moral reasoning—the ethical equivalent of learning to crawl—is to weigh other?s interests against one?s own. This in turn requires sympathy and imagination: without which there is no capacity for moral thought. To see an animal in pain is enough, for most, to engage sympathy.35)When that happens, it is not a mistake: it is mankind?s instinct for moral reasoning in action, an instinct that should be encouraged rather than laughed at. Section ? Writing (15 points) 36. Directions: A. Study the following set of pictures carefully and write an essay in no less than 120 words. B. Your essay must be written clearly on ANSWER SHEET 2. C. Your essay should cover all the information provided and meet the requirements below: 1. Interpret the following pictures. 2. Predict the tendency of tobacco consumption and give your reasons. Section ? Writing(15 points) 一、审题谋篇 1997年的作文看起来非常简单,是我们非常熟悉的吸烟问题,似乎每个人都能就此发表自己的观点。实 则不然,这篇文章有几个暗含的难点。首先,与往年不同,本年度的图表比较复杂,包含四幅图画。第一幅 图是1994年与1995年世界烟草总产量的一个比较图,第二幅图是一个扇形图,说明烟民占世界人口的比例,在 此,我们可以看到这个比例是非常高的,言外之意,对吸烟的控制将不是一件轻而易举的事情,,第三幅图和 第四幅图分别显示吸烟带来的经济损失和人员损失之大,进一步说明控制吸烟势在必行。考生看了四幅图后, 首先应该做的一件事情就是寻找四幅图之间的逻辑联系和内在线索,而不是简单地依次描述四幅图。其次, 本作文另外一个难点就是文章的重点或落脚点比较隐蔽,也就是在文章提纲的第二点要求上:预测烟草消费 的趋势并给出原因。 基于以上的分析,这篇文章最好分为两段或者三段。第一段段首可以着重利用后两幅图,论述吸烟的危 害性,段末部分使用第一幅图点出文章的中心:烟草消费量可能下降的趋势。文章第二段可着重论述这种消 费下降趋势的原因。考生也可以在第二段段首点明烟草消费下降的趋势,进而阐述原因。范文使用第二种方 法。考生可以根据实际情况,增加一个第三段,客观地分析烟草消费量下降不是一件容易的事,在此可以利 用第三幅图,说明烟民占世界总人口的比例较大,控制吸烟不是一件容易的事情。 二、参考范文 It goes without saying that tobacco consumption and tobacco industry have always been a hot-debated issue in our society. With the increasing scope of the tobacco industry, more and more people become addicted to smoking. The tremendous tobacco consumption not only causes vast economic losses but also threatens people?s health. The annual economic loss due to tobacco consumption amounts to 200 billion US dollars, and meanwhile 3 million people worldwide lose their lives because of smoking-related diseases. To our great relief, there is a tendency of declining in tobacco consumption—the year between 1994 and 1995, as indicated by chart 1, witnessed a drop in total tobacco production in the world, declining from 14.364 million pounds in 1994 to 14.2 million pounds in 1995. The reason that more and more people begin to quit smoking is mainly people?s increasing awareness of the harmful effects of tobacco. If this trend can be maintained, the number of people dying of smoking-related diseases can be significantly reduced. Therefore, great efforts should be made by people in all walks of the society to stop smoking. Needless to say, no easy solution can be found to solve the smoking problem. To begin with, the percentage of smokers is too large: according to a survey, there are 1.2 billion tobacco consumers in the world, accounting for approximately 20 percent of the world population. It wouldn?t be an easy task to persuade such a large amount of people to give up their smoking habits, especially the heavy smokers. In addition, tobacco industry still plays a very important role in the development of the economy. As a consequence, only with concerted efforts of all people, can this problem be properly dealt with. 三、范文点评 文章结构: 从结构上看,该范文对提纲做出适当调整,分为三段而不是两段。第一段采用了先总后分的写作方法, 段首先对图画做出总体评价,进而指出烟草业和吸烟对人类社会的危害。第二段段首和上段形成对比,段首 指出这种现象有所改变:烟草消费数量有所下降,进而结合图表,指出这种下降的原因何在。第三段采用总 —分—总的写作方法,段首为主题句,指出吸烟问题解决起来并没有那么容易,段落中使用To begin with和 In addition从两个方面进行论证,段尾对本段进行了总结。 语言亮点: 1. hot-debated issue:“讨论的热点”,还可以说widely-discussed issue。 2. become addicted to:或be addicted to上瘾。例如:More and more young people become addicted to on-line chatting.,越来越多的年轻人着迷于网上聊天,。 3. annual:每年的,近义词为yearly。Every year为副词,相当于annually。 4. due to:“因为”,如:Most accidents are due to driving at high speed.,大部分事故是由于高速驾驶造成的,。类似的表达有:because of, on account of, as result of, in consequence of, caused by等。 5. amount to:“达到,某数量,,到…之多”。这是一个图表题中经常使用的结构,例如:Expenses on food amount to 30% of every Chinese family?s income.,用于食物的开支占每个中国家庭收入的30%,。类似的表达有:add up to, come to, total, sum up等。 6. meanwhile:与此同时。词性为副词,相当于at the same time。 7. smoking-related diseases:与吸烟有关的疾病。 8.,much,to one?s great relief:使某人大为放心,使某人深感宽慰。 9. witness:“时间+experience /see /witness +变化+在哪方面”可用来表达某段时间发生的变化。 10. The reason that... is... :用reason接从句表原因有两种方式:一种是范文中的用法the reason ,that,,定语从句,... is,that,...,另一种是the reason why,接同位语从句,... is that...,如:The reason why we cook food much faster today is that the changes in food preparation methods,今天我们烹饪食物的速度提高了很多的原因是在准备食物的方法上有了变化,。 11. maintain:保持。例如:If the present rate of economic increase can be maintained, the majority of Chinese will live a well-off life in the middle of the 21st century.,如果中国经济增长能保持目前的速度,21世纪中叶,大部分中国人就能过上小康生活,。 12. in all walks of the society:“社会的各个方面”,又如:men in all walks of life,各行各业的人,。 13. To begin with:首先,相当于first of all或first(ly)。 14. account for:,在数量、比例方面,占。例如:Farmers account for about 80% of the total population of China. ,农民占中国总人口的约80%。, 15. In addition:而且,再者。相当于What?s more。 16. concerted:“商定的,共同计划或完成的”,如:We made a concerted effort to solve the problem.,我们一起努力解决了这个问题, 四、写作误区 篇章结构误区: 生硬的图表罗列是本题写作的第一个误区。任何形式的图表作文,考试的目的都不是单纯的描写,而是要抓住图表之间的内在联系。如果只是罗列现象、描写图表,就会犯言之无物的错误,难以展开有力有效的分析。 本题写作的第二个误区是跑题,因为很多考生一看到题目,就理所当然地认为这是非常熟悉的“吸烟有害健康”主题的文章,便开始大谈吸烟的危害、主动吸烟、被动吸烟,及至如何降低吸烟对社会及个人造成的危害,等等。考生犯这样的错误说明没有审清楚题,题目的要求有两点:解释图片,预测烟草消费的趋势并给出原因。换句话说,这样的考生没有意识到或者根本没有看清楚第二个要求。 语言表达错误: ?用词不当: The other reason is that smoking is forbidden in the popular areas in more and more countries.,The other reason is that smoking is forbidden in public areas in more and more countries., Thus I think the tendency of world tobacco consumption is descend.,Thus I think the tendency of world tobacco consumption is to decrease., ?动词误用: Even the strongest method can?t decline the number of smokers.,Even the toughest method can?t make the number of smokers decline., ?连词误用: Because of the tendency of tobacco consumption, I think the number of cigarette smokers is rising in some developing countries.,As far as the tendency of tobacco consumption is concerned, I think the number of cigarette smokers is rising in some developing countries., ?冠词错误: For above-mentioned reasons, I believe the consumption of tobacco will decrease and all problems it causes will be solved.,For all the above-mentioned reasons, I believe the consumption of tobacco will decrease and all the problems it causes will be solved., ?主谓搭配错误: More and more people believe smoking do harm to health, waste money.,More and more people believe smoking does harm to health, and is a waste of money., ?中式英语: From the pictures, we first know that there are a lot of smoking people in the world.,From the pictures, we first know that there are a lot of smokers in the world., ?句意笼统: You can see the number from the chart, which make you think deeply of the harm of smoking. ,It can be seen from the chart that there are 200 billion dollars? loss and 3 million deaths resulting from smoking., 1998年全真试题 Section I Cloze Test Directions: For each numbered blank in the following passage, there are four choices marked [A], [B], [C], and [D]. Choose the best one and mark your answer on ANSWER SHEET 1 by blackening the corresponding letter in the brackets with a pencil. (10 points) Until recent l y most historians spoke very critically of the Industrial Revolution. They1that in the long run industrialization greatly raised the standard of living for the 2 man. But they insisted that its 3 results during the period from 1750 to 1850 were widespread poverty and misery for the 4 of the English population. 5 contrast, they saw in the preceding hundred years from 1650 to 1750, when England was still a 6 agricultural country, a period of great abundance and prosperity. This view, 7 , is generally thought to be wrong. Specialists 8 history and economics, have 9 two things: that the period from 1650 to 1750 was 10 by great poverty, and that industrialization certainly did not worsen and may have actually improved the conditions for the majority of the populace. 1. ,A,admitted ,B,believed ,C,claimed ,D,predicted 2. ,A,plain ,B,average ,C,mean ,D,normal 3. ,A,momentary ,B,prompt ,C,instant ,D,immediate 4. ,A,bulk ,B,host ,C,gross ,D,magnitude 5. ,A,On ,B,With ,C,For ,D,By 6. ,A,broadly ,B,thoroughly ,C,generally ,D,completely 7. ,A,however ,B,meanwhile ,C,therefore ,D,moreover 8. ,A,at ,B,in ,C,about ,D,for 9. ,A,manifested ,B,approved ,C,shown ,D,speculated 10.,A,noted ,B,impressed ,C,labeled ,D,marked Section ? Reading Comprehension Directions: Each of the passages below is followed by some questions. For each question there are four answers marked [A], [B], [C] and [D]. Read the passages carefully and choose the best answer to each of the questions. Then mark your answer on the ANSWER SHEET 1 by blackening the corresponding letter in the brackets. (40 points) Text 1 Few creations of big technology capture the imagination like giant dams. Perhaps it is humankind?s long suffering at the mercy of flood and drought that makes the idea of forcing the waters to do our bidding so fascinating. But to be fascinated is also, sometimes, to be blind. Several giant dam projects threaten to do more harm than good. The lesson from dams is that big is not always beautiful. It doesn?t help that building a big, powerful dam has become a symbol of achievement for nations and people striving to assert themselves. Egypt?s leadership in the Arab world was cemented by the Aswan High Dam. Turkey?s bid for First World status includes the giant Ataturk Dam. But big dams tend not to work as intended. The Aswan Dam, for example, stopped the Nile flooding but deprived Egypt of the fertile silt that floods left -- all in return for a giant reservoir of disease which is now so full of silt that it barely generates electricity. And yet, the myth of controlling the waters persists. This week, in the heart of civilized Europe, Slovaks and Hungarians stopped just short of sending in the troops in their contention over a dam on the Danube. The huge complex will probably have all the usual problems of big dams. But Slovakia is bidding for independence from the Czechs, and now needs a dam to prove itself. Meanwhile, in India, the World Bank has given the go-ahead to the even more wrong-headed Narmada Dam. And the bank has done this even though its advisors say the dam will cause hardship for the powerless and environmental destruction. The benefits are for the powerful, but they are far from guaranteed. Proper, scientific study of the impacts of dams and of the cost and benefits of controlling water can help to resolve these conflicts. Hydroelectric power and flood control and irrigation are possible without building monster dams. But when you are dealing with myths, it is hard to be either proper, or scientific. It is time that the world learned the lessons of Aswan. You don?t need a dam to be saved. 11. The third sentence of Paragraph 1 implies that ________. [A] people would be happy if they shut their eyes to reality [B] the blind could be happier than the sighted [C] over-excited people tend to neglect vital things. [D] fascination makes people lose their eyesight 12. In Paragraph 5, “the powerless” probably refers to ________. [A] areas short of electricity [B] dams without power stations [C] poor countries around India [D] common people in the Narmada Dam area 13.What is the myth concerning giant dams? [A] They bring in more fertile soil. [B] They help defend the country. [C] They strengthen international ties. [D] They have universal control of the waters. 14.What the author tries to suggest may best be interpreted as ________. [A] “It?s no use crying over spilt milk” [B] “More haste, less speed” [C] “Look before you leap” [D] “He who laughs last laughs best” Text 2 Well, no gain without pain, they say. But what about pain without gain? Everywhere you go in America, you hear tales of corporate revival. What is harder to establish is whether the productivity revolution that businessmen assume they are presiding over is for real. The official statistics are mildly discouraging. They show that, if you lump manufacturing and services together, productivity has grown on average by 1.2% since 1987. That is somewhat faster than the average during the previous decade. And since 1991, productivity has increased by about 2% a year, which is more than twice the 1978-87 average. The trouble is that part of the recent acceleration is due to the usual rebound that occurs at this point in a business cycle, and so is not conclusive evidence of a revival in the underlying trend. There is, as Robert Rubin, the treasury secretary, says, a “disjunction” between the mass of business anecdote that points to a leap in productivity and the picture reflected by the statistics. Some of this can be easily explained. New ways of organizing the workplace -- all that re-engineering and downsizing -- are only one contribution to the overall productivity of an economy, which is driven by many other factors such as joint investment in equipment and machinery, new technology, and investment in education and training. Moreover, most of the changes that companies make are intended to keep them profitable, and this need not always mean increasing productivity: switching to new markets or improving quality can matter just as much. Two other explanations are more speculative. First, some of the business restructuring of recent years may have been ineptly done. Second, even if it was well done, it may have spread much less widely than people suppose. Leonard Schlesinger, a Harvard academic and former chief executive of Au Bong Pain, a rapidly growing chain of bakery cafes, says that much “re-engineering” has been crude. In many cases, he believes, the loss of revenue has been greater than the reductions in cost. His colleague, Michael Beer, says that far too many companies have applied re-engineering in a mechanistic fashion, chopping out costs without giving sufficient thought to long-term profitability. BBDO?s Al Rosen shine is blunter. He dismisses a lot of the work of re-engineering consultants as mere rubbish -- “the worst sort of ambulance chasing.” 15. According to the author, the American economic situation is ________. [A] not as good as it seems [B] at its turning point [C] much better than it seems [D] near to complete recovery 16. The official statistics on productivity growth ________. [A] exclude the usual rebound in a business cycle [B] fall short of businessmen?s anticipation [C] meet the expectation of business people [D] fail to reflect the true state of economy 17. The author raises the question “what about pain without gain?” because ________. [A] he questions the truth of “no gain without pain” [B] he does not think the productivity revolution works [C] he wonders if the official statistics are misleading [D] he has conclusive evidence for the revival of businesses 18. Which of the following statements is NOT mentioned in the passage? [A] Radical reforms are essential for the increase of productivity. [B] New ways of organizing workplaces may help to increase productivity. [C] The reduction of costs is not a sure way to gain long-term profitability. [D] The consultants are a bunch of good-for-nothings. Text 3 Science has long had an uneasy relationship with other aspects of culture. Think of Gallileo’s 17th-century trial for his rebelling belief before the Catholic Church or poet William Blake?s harsh remarks against the mechanistic worldview of Isaac Newton. The schism between science and the humanities has, if anything, deepened in this century. Until recently, the scientific community was so powerful that it could afford to ignore its critics -- but no longer. As funding for science has declined, scientists have attacked “anti-science” in several books, notably Higher Superstition, by Paul R. Gross, a biologist at the University of Virginia, and Norman Levitt, a mathematician at Rutgers University; and The Demon-Haunted World, by Carl Sagan of Cornell University. Defenders of science have also voiced their concerns at meetings such as “The Flight from Science and Reason,” held in New York City in 1995, and “Science in the Age of (Mis) information,” which assembled last June near Buffalo. Anti-science clearly means different things to different people. Gross and Levitt find fault primarily with sociologists, philosophers and other academics who have questioned science?s objectivity. Sagan is more concerned with those who believe in ghosts, creationism and other phenomena that contradict the scientific worldview. A survey of news stories in 1996 reveals that the anti-science tag has been attached to many other groups as well, from authorities who advocated the elimination of the last remaining stocks of smallpox virus to Republicans who advocated decreased funding for basic research. Few would dispute that the term applies to the Unabomber, whose manifesto, published in 1995, scorns science and longs for return to a pre-technological utopia. But surely that does not mean environmentalists concerned about uncontrolled industrial growth are anti-science, as an essay in US News & World Report last May seemed to suggest. The environmentalists, inevitably, respond to such critics. The true enemies of science, argues Paul Ehrlich of Stanford University, a pioneer of environmental studies, are those who question the evidence supporting global warming, the depletion of the ozone layer and other consequences of industrial growth. Indeed, some observers fear that the anti-science epithet is in danger of becoming meaningless. “The term „anti-science? can lump together too many, quite different things,” notes Harvard University philosopher Gerald Holton in his 1993 work Science and Anti-Science. “They have in common only one thing that they tend to annoy or threaten those who regard themselves as more enlightened.” 19. The word “schism” (Line 4, Paragraph 1) in the context probably means ________. [A] confrontation [B] dissatisfaction [C]separation [D]contempt 20. Paragraphs 2 and 3 are written to ________. [A]discuss the cause of the decline of science?s power [B]show the author?s sympathy with scientists [C]explain the way in which science develops [D]exemplify the division of science and the humanities 21. Which of the following is true according to the passage? [A] Environmentalists were blamed for anti-science in an essay. [B] Politicians are not subject to the labeling of anti-science. [C] The “more enlightened” tend to tag others as anti-science. [D] Tagging environmentalists as “anti-science” is justifiable. 22. The author?s attitude toward the issue of “science vs. anti-science” is ________. [A] impartial [B] subjective [C] biased [D] puzzling Text 4 Emerging from the 1980 census is the picture of a nation developing more and more regional competition, as population growth in the Northeast and Midwest reaches a near standstill. This development -- and its strong implications for US politics and economy in years ahead -- has enthroned the South as America?s most densely populated region for the first time in the history of the nation?s head counting. Altogether, the US population rose in the 1970s by 23.2 million people -- numerically the third-largest growth ever recorded in a single decade. Even so, that gain adds up to only 11.4 percent, lowest in American annual records except for the Depression years. Americans have been migrating south and west in larger numbers since World War II, and the pattern still prevails. Three sun-belt states -- Florida, Texas and California -- together had nearly 10 million more people in 1980 than a decade earlier. Among large cities, San Diego moved from 14th to 8th and San Antonio from 15th to 10th -- with Cleveland and Washington. D. C., dropping out of the top 10. Not all that shift can be attributed to the movement out of the snow belt, census officials say. Nonstop waves of immigrants played a role, too -- and so did bigger crops of babies as yesterday?s “baby boom” generation reached its child bearing years. Moreover, demographers see the continuing shift south and west as joined by a related but newer phenomenon: More and more, Americans apparently are looking not just for places with more jobs but with fewer people, too. Some instances— ?Regionally, the Rocky Mountain states reported the most rapid growth rate -- 37.1 percent since 1970 in a vast area with only 5 percent of the US population. ?Among states, Nevada and Arizona grew fastest of all: 63.5 and 53.1 percent respectively. Except for Florida and Texas, the top 10 in rate of growth is composed of Western states with 7.5 million people -- about 9 per square mile. The flight from overcrowdedness affects the migration from snow belt to more bearable climates. Nowhere do 1980 census statistics dramatize more the American search for spacious living than in the Far West. There, California added 3.7 million to its population in the 1970s, more than any other state. In that decade, however, large numbers also migrated from California, mostly to other parts of the West. Often they chose -- and still are choosing -- somewhat colder climates such as Oregon, Idaho and Alaska in order to escape smog, crime and other plagues of urbanization in the Golden State. As a result, California?s growth rate dropped during the 1970s, to 18.5 percent -- little more than two thirds the 1960s? growth figure and considerably below that of other Western states. 23. Discerned from the perplexing picture of population growth the 1980 census provided, America in 1970s ________. [A] enjoyed the lowest net growth of population in history [B] witnessed a southwestern shift of population [C] underwent an unparalleled period of population growth [D] brought to a standstill its pattern of migration since World War II 24. The census distinguished itself from previous studies on population movement in that ________. [A] it stresses the climatic influence on population distribution [B] it highlights the contribution of continuous waves of immigrants [C] it reveals the Americans? new pursuit of spacious living [D] it elaborates the delayed effects of yesterday?s “baby boom” 25. We can see from the available statistics that ________. [A] California was once the most thinly populated area in the whole US [B] the top 10 states in growth rate of population were all located in the West [C] cities with better climates benefited unanimously from migration [D] Arizona ranked second of all states in its growth rate of population 26. The word “demographers” (Line 1, Paragraph 8) most probably means ________. [A] people in favor of the trend of democracy [B] advocates of migration between states [C] scientists engaged in the study of population [D] conservatives clinging to old patterns of life Text 5 Scattered around the globe are more than 100 small regions of isolated volcanic activity known to geologists as hot spots. Unlike most of the world?s volcanoes, they are not always found at the boundaries of the great drifting plates that make up the earth?s surface; on the contrary, many of them lie deep in the interior of a plate. Most of the hot spots move only slowly, and in some cases the movement of the plates past them has left trails of dead volcanoes. The hot spots and their volcanic trails are milestones that mark the passage of the plates. That the plates are moving is now beyond dispute. Africa and South America, for example, are moving away from each other as new material is injected into the sea floor between them. The complementary coastlines and certain geological features that seem to span the ocean are reminders of where the two continents were once joined. The relative motion of the plates carrying these continents has been constructed in detail, but the motion of one plate with respect to another cannot readily be translated into motion with respect to the earth?s interior. It is not possible to determine whether both continents are moving in opposite directions or whether one continent is stationary and the other is drifting away from it. Hot spots, anchored in the deeper layers of the earth, provide the measuring instruments needed to resolve the question. From an analysis of the hot-spot population it appears that the African plate is stationary and that it has not moved during the past 30 million years. The significance of hot spots is not confined to their role as a frame of reference. It now appears that they also have an important influence on the geophysical processes that propel the plates across the globe. When a continental plate come to rest over a hot spot, the material rising from deeper layers creates a broad dome. As the dome grows, it develops deep fissures (cracks); in at least a few cases the continent may break entirely along some of these fissures, so that the hot spot initiates the formation of a new ocean. Thus just as earlier theories have explained the mobility of the continents, so hot spots may explain their mutability (inconstancy). 27. The author believes that ________. [A] the motion of the plates corresponds to that of the earth?s interior [B] the geological theory about drifting plates has been proved to be true [C] the hot spots and the plates move slowly in opposite directions [D] the movement of hot spots proves the continents are moving apart 28. That Africa and South America were once joined can be deduced from the fact that ________. [A] the two continents are still moving in opposite directions [B] they have been found to share certain geological features [C] the African plate has been stable for 30 million years [D] over 100 hot spots are scattered all around the globe 29. The hot spot theory may prove useful in explaining ________. [A] the structure of the African plates [B] the revival of dead volcanoes [C] the mobility of the continents [D] the formation of new oceans 30. The passage is mainly about ________. [A] the features of volcanic activities [B] the importance of the theory about drifting plates [C] the significance of hot spots in geophysical studies [D] the process of the formation of volcanoes Section IV English-Chinese Translation Directions: Read the following passage carefully and then translate the underlined sentences into Chinese. Your translation must be written clearly on the ANSWER SHEET 2. (15 points) They were, by far, the largest and most distant objects that scientists had ever detected: a strip of enormous cosmic clouds some 15 billion light-years from earth. 31) But even more important, it was the farthest that scientists had been able to look into the past, for what they were seeing were the patterns and structures that existed 15 billion years ago. That was just about the moment that the universe was born. What the researchers found was at once both amazing and expected: the US National Aeronautics and Space Administration?s Cosmic Background Explorer satellite -- Cobe -- had discovered landmark evidence that the universe did in fact begin with the primeval explosion that has become known as the Big Bang (the theory that the universe originated in an explosion from a single mass of energy). 32) The existence of the giant clouds was virtually required for the Big Bang, first put forward in the 1920s, to maintain its reign as the dominant explanation of the cosmos. According to the theory, the universe burst into being as a submicroscopic, unimaginably dense knot of pure energy that flew outward in all directions, emitting radiation as it went, condensing into particles and then into atoms of gas. Over billions of years, the gas was compressed by gravity into galaxies, stars, plants and eventually, even humans. Cobe is designed to see just the biggest structures, but astronomers would like to see much smaller hot spots as well, the seeds of local objects like clusters and superclusters of galaxies. They shouldn?t have long to wait. 33) Astrophysicists working with ground-based detectors at the South Pole and balloon-borne instruments are closing in on such structures, and may report their findings soon. 34) If the small hot spots look as expected, that will be a triumph for yet another scientific idea, a refinement of the Big Bang called the inflationary universe theory. Inflation says that very early on, the universe expanded in size by more than a trillion trillion trillion trillion fold in much less than a second, propelled by a sort of antigravity. 35) Odd though it sounds, cosmic inflation is a scientifically plausible consequence of some respected ideas in elementary particle physics, and many astrophysicists have been convinced for the better part of a decade that it is true. 31. ________ 32. ________ 33. ________ 34. ________ 35. ________ Section V Writing Directions: [A] Study the following cartoon carefully and write an essay in no less than 150 words. [B] Your essay must be written clearly on the ANSWER SHEET 2. (15 points) [C] Your essay should meet the requirements below: 1. Write out the messages conveyed by the cartoon. 2. Give your comments. Section V Writing 一、审题谋篇 本题为漫画加提纲式作文。文章的主题应该落在讽刺虚假承诺上,即:承诺自己份内之事这一不良现象。 考生在审题时,从漫画和旁边的打油诗可以看出该文章的关键词为虚假承诺,false promise或empty promise,。提纲有两点要求:写出漫画所要表达的信息,进行评论。由此可见,这是一篇描写加议论的文章。 文章第一段可以从描写漫画入手。漫画题材的写作,应着重抓住漫画本身所传达的信息,即漫画的寓意,对 漫画本身不用拘泥于细节。考生可以在段尾点明该漫画所要表达的信息:虚假承诺。由于提纲的第二点要求 比较泛,因此,考生可以根据各自的思路展开。第二段可以针对主题进一步讨论,既可以列举社会上众多的 虚假承诺的不良现象,也可以着重讨论虚假承诺的危害,亦或追寻虚假承诺的根源。在第三段中考生可提出 自己的观点,即,解决该问题的方法。 二、参考范文 Whoever sees this cartoon can?t help laughing. How funny it is-A hen is holding a notice promising that her eggs would be round and surely would contain everything a normal egg contains. But at second thought, it is anything but funny because the picture reveals a prevalent phenomenon in our society: Many of these promises are meaningless since the promise provided is nothing but the normal responsibility of the promise maker. Odd and funny as they sound,such false promises are so prevalent that we cannot afford to ignore them. Manufacturing units guarantee to turn out products of good quality; commercial enterprises swear to provide genuine commodities and enthusiastic services; administration departments assure to perform their tasks effectively and fairly without taking any bribery. As a matter of fact,these cannot be called promises at all since they are no more than their normal duties and obligations. It seems that we should be grateful to them because they just do what they are supposed to do. I dare say these empty promises are causing great damage to our society both materially and morally. But the hen and her likes should know that by dishonest words no one could survive the intense competition under market economy system. People engaged in providing all kinds of services should honestly inform their customers of the service that they can enjoy because mutual trust is the foundation of business. They should remember the old saying, "Honesty is the best policy. “On the other hand, necessary laws and regulations should be adopted and enforced to ensure a trust worthy economic and social environment. Only in this way, can both parties, service providers and those served, enjoy a healthy atmosphere of mutual trust. 三、范文点评 文章结构: 从结构上看,该范文对提纲做出适当调整,分为三段而不是两段。第一段采用了先总后分再总的写作方 法,段首先用Whoever sees this cartoon can?t help laughing一句话对图画做出总体评价,进而描写图画,在段 末指出:这幅画实际并不可笑,而是讽刺了社会上非常普遍的虚假承诺现象。这句话也是文章的中心所在。 第二段段首承接上段,指出这种现象不容忽视,进而使用列举的方法举了三个例子,说明其普遍性并与第一 段呼应,指出这些现象都是在承诺本职所在的工作。第三段段首为主体句,段落中从两个方面提出改变这种 现象的措施,段尾进行总结,与段首形成呼应。 语言亮点: 1. whoever:是疑问代词who的强调形式,也可做关系代词,等于any person who,如:Whoever comes is welcome.,任何人来了都欢迎,。同类的词还有:whatever=anything that,无论什么,, whichever=any one that ,任何一个,。 2. can?t help laughing:can?t help doing 忍不住干某事。如:We can?t help worrying about our air condition when watching so many cars pumping huge amounts of waste gases into the atmosphere every day.,看到每天都有这么多 汽车排放大量废气到大气中,我们禁不住为我们的空气质量而担忧,。 3. at second thought:仔细一想,at first thought乍一想。 4. anything but:根本不,决不,远非。例如:The present situation is anything but a relief.,目前形势绝对不容乐观,。 5. prevalent:"普遍的",同义词为:widespread, common, predominant等。 6. nothing but和no more than:仅仅,只不过。类似的表达有:amounts to nothing but。 7. as:句子的表语提到句首时,as引导的是让步状语从句,如:Popular as it is, in terms of nutrition, fast food is far from satisfactory.,虽然快餐食品很流行,但从营养上来说,它很难让人满意,。 8. afford:经得起。例如:Our country cannot afford to go through another Cultural Revolution.,我们国家经不起再来一次文化大革命,。 9. ignore:"忽视,忽略,对…视而不见"。同义词有:disregard, overlook, look over等。 10. guarantee,保证,、swear,发誓,、assure,保证,:三个动词词义相近,形成对仗。同义词还有:promise, vow, pledge, give a warranty。 11. likes:类似的,人或物,。如:I?ve never seen the likes of this before.,我从没见过这样的事,。 12. inform sb. of sth. :通知某人某事。该结构还经常用于被动形式,如The news media keep us informed of what is happening in our country and in the world.,新闻媒体使我们知道我国和世界发生了什么事情,。 13. adopted:采纳,采用。近义词有:introduced。 14. party:,缔约或谈判等的,一方,如:In the agreement all parties guaranteed to take their own responsibilities. ,在 协议 离婚协议模板下载合伙人协议 下载渠道分销协议免费下载敬业协议下载授课协议下载 中各方都承诺担负起他们各自的责任,。 15. a trustworthy economic and social environment:一个值得信赖的经济和社会环境。 四、写作误区 篇章结构误区: 1998年的作文第一个需要避免的问题还是跑题。首先,有些考生只注意到广告失实,并未抓住 "承诺"二字,于是将文章主旨说成伪劣产品或虚假广告,结果跑题了。其次,考生在下笔前对文章的结构要心中有数,切不可想到什么就写什么,否则会造成文章思路不清,结构混乱。与往年试题不同,本年度的作文是一副漫画,为了要表达作者的意图,图画往往会运用夸张等手段,因此考生在描写图画时,切不可拘泥于个别单词或语句的表达,对于"见棱见角","蛋白","蛋黄"等陌生词汇,可采用迂回概括的方法表达思想。 语言表达错误: ?词性错误: These dishonest behaviors, if permitted to continue, will sure do harm to the develop of our country.,These dishonest behaviors, if permitted to continue, will surely do harm to the development of our country., ?词义错误: It is taken for granted that an egg is round and has three parts.,It is known to all that an egg is round and has three parts., ?捏造词汇: Promise can generate worse results than unpromise if customers find they are cheated at last.,Making false promises can bring about worse results than making no promise if customers find they are cheated at last., ?结构不平衡: Nowadays, making promises is prevalent in society, from companies, factories, to shops.,Nowadays, making promises is prevalent in society, from companies and factories, to shops and stores., ?非谓语结构错误: For one thing, making promises about one?s products is so popular that no one wants to leave behind.,For one thing, making promises about one?s products is so popular that no one wants to be left behind., ?指代不清: The advertisers make false promises because they will buy more goods.,The advertisers make false promises because the consumers will be induced to buy more goods., ?关联词误用: Promises are necessary and we need not false and cheaty promises.,Promises are necessary but we need not false and deceptive ones., ?中式英语: My solutions to the problem have several ways.,As far as I am concerned, there are several solutions to the problem., ?句子含义不完整: Those who intend to deceive consumers should be punished as well as consumers consciousness.,Those who intend to deceive consumers should be punished and consumers should be on their guard against any false promises., ?句子不连贯: So to solve this problem, it needs everyone to make great effort.(It needs everyone?s efforts to solve this problem.) 1999年全真试题 Part ?Cloze Test Directions: For each numbered blank in the following passage, there are four choices marked ,A,, ,B,, ,C, and ,D,. Choose the best one and mark your answer on ANSWER SHEET 1 by blackening the corresponding letter in the brackets with a pencil. (10 points) Industrial safety does not just happen. Companies 1 low accident rates plan their safety programs, work hard to organize them, and continue working to keep them 2 and active. When the work is well done, a 3 of accident free operations is established 4 time lost due to injuries is kept at a minimum. Successful safety programs may 5 greatly in the emphasis placed on certain aspects of the program. Some place great emphasis on mechanical guarding. Others stress safe work practices by 6 rules or regulations. 7 others depend on an emotional appeal to the worker. But, there are certain basic ideas that must be used in every program if maximum results are to be obtained. There can be no question about the value of a safety program. From a financial standpoint alone, safety 8 . The fewer the injury 9 , the better the workman?s insurance rate. This may mean the difference between operating at 10 or at a loss. 1.,A,at ,B,in ,C,on ,D,with 2.,A,alive ,B,vivid ,C,mobile ,D,diverse 3.,A,regulation ,B,climate ,C,circumstance ,D, requirement 4.,A,where ,B,how ,C,what ,D,unless 5.,A,alter ,B,differ ,C,shift ,D,distinguish 6.,A,constituting ,B,aggravating ,C,observing ,D,justifying 7.,A,Some ,B,Many ,C,Even ,D, Still 8.,A,comes off ,B,turns up ,C,pays off ,D, holds up 9.,A,claims ,B,reports ,C,declarations ,D,proclamations 10.,A,an advantage ,B,a benefit ,C,an interest ,D,a profit Part ?Reading Comprehension Directions: Each of the passages below is followed by some questions. For each question there are four answers marked,A,, ,B,,,C,and,D,. Read the passages carefully and choose the best answer to each of the questions. Then mark your answer on ANSWER SHEET 1 by blackening the corresponding letter in the brackets with a pencil. (40 points) Passage 1 It?s a rough world out there. Step outside and you could break a leg slipping on your doormat. Light up the stove and you could burn down the house. Luckily, if the doormat or stove failed to warn of coming disaster, a successful lawsuit might compensate you for your troubles. Or so the thinking has gone since the early 1980s, when juries began holding more companies liable for their customers? misfortunes. Feeling threatened, companies responded by writing ever longer warning labels, trying to anticipate every possible accident. Today, stepladders carry labels several inches long that warn, among other things, that you might—surprise!—fall off. The label on a child?s Batman cape cautions that the toy “does not enable user to fly”. While warnings are often appropriate and necessary—the dangers of drug interactions, for example—and many are required by state or federal regulations, it isn?t clear that they actually protect the manufacturers and sellers from liability if a customer is injured. About 50 percent of the companies lose when injured customers take them to court. Now the tide appears to be turning. As personal injury claims continue as before, some courts are beginning to side with defendants, especially in cases where a warning label probably wouldn?t have changed anything. In May, Julie Nimmons, president of Schutt Sports in Illinois, successfully fought a lawsuit involving a football player who was paralyzed in a game while wearing a Schutt helmet. “We?re really sorry he has become paralyzed, but helmets aren?t designed to prevent those kinds of injuries, ” says Nimmons. The jury agreed that the nature of the game, not the helmet, was the reason for the athlete?s injury. At the same time, the American Law Institute—a group of judges, lawyers, and academics whose recommendations carry substantial weight—issued new guidelines for tort law stating that companies need not warn customers of obvious dangers or bombard them with a lengthy list of possible ones. “Important information can get buried in a sea of trivialities, ” says a law professor at Cornell Law School who helped draft the new guidelines. If the moderate end of the legal community has its way, the information on products might actually be provided for the benefit of customers and not as protection against legal liability. 11. What were things like in 1980s when accidents happened? ,A,Customers might be relieved of their disasters through lawsuits. ,B,Injured customers could expect protection from the legal system. ,C,Companies would avoid being sued by providing new warnings. ,D,Juries tended to find fault with the compensations companies promised. 12. Manufacturers as mentioned in the passage tend to. ,A,satisfy customers by writing long warnings on products ,B,become honest in describing the inadequacies of their products ,C,make the best use of labels to avoid legal liability ,D,feel obliged to view customers? safety as their first concern 13. The case of Schutt helmet demonstrated that. ,A,some injury claims were no longer supported by law ,B,helmets were not designed to prevent injuries ,C,product labels would eventually be discarded ,D,some sports games might lose popularity with athletes 14. The author?s attitude towards the issue seems to be. ,A,biased ,B,indifferent ,C,puzzling ,D,objective Passage 2 In the first year or so of Web business, most of the action has revolved around efforts to tap the consumer market. More recently, as the Web proved to be more than a fashion, companies have started to buy and sell products and e sense because business people typically know what product they?re looking for. Nonetheless, many companies still hesitate to use the Web because of doubts about its reliability. “Businesses need to feel they can trust the pathway between them and the supplier,” says senior analyst Blane Erwin of Forrester Research. Some companies are limiting the risk by conducting online transactions only with established business partners who are given access to the company?s private intranet. Another major shift in the model for Internet commerce concerns the technology available for marketing. Until recently, Internet marketing activities have focused on strategies to “pull” customers into sites. In the past year, however, software companies have developed tools that allow companies to “push” information directly out to consumers, transmitting marketing messages directly to targeted customers. Most notably, the Pointcast Network uses a screen saver to deliver a continually updated stream of news and advertisements to subscribers? computer monitors. Subscribers can customize the information they want to receive and proceed directly to a company?s Web site. Companies such as Virtual Vineyards are already starting to use similar technologies to push messages to customers about special sales, product offerings, or other events. But push technology has earned the contempt of many Web users. Online culture thinks highly of the notion that the information flowing onto the screen comes there by specific request. Once commercial promotion begins to fill the screen uninvited, the distinction between the Web and television fades. That?s a prospect that horrifies Net purists. But it is hardly inevitable that companies on the Web will need to resort to push strategies to make money. The examples of Virtual Vineyards, Amazon .com, and other pioneers show that a Web site selling the right kind of products with the right mix of interactivity, hospitality, and security will attract online customers. And the cost of computing power continues to free fall, which is a good sign for any enterprise setting up shop in silicon. People looking back 5 or 10 years from now may well wonder why so few companies took the online plunge. 15. We learn from the beginning of the passage that Web business. ,A,has been striving to expand its market ,B,intended to follow a fanciful fashion ,C,tried but in vain to control the market ,D,has been booming for one year or so 16. Speaking of the online technology available for marketing, the author implies that. ,A,the technology is popular with many Web users ,B,businesses have faith in the reliability of online transactions ,C,there is a radical change in strategy ,D,it is accessible limitedly to established partners 17. In the view of Net purists, . ,A,there should be no marketing messages in online culture ,B,money making should be given priority to on the Web ,C,the Web should be able to function as the television set ,D,there should be no online commercial information without requests 18. We learn from the last paragraph that. ,A,pushing information on the Web is essential to Internet commerce ,B,interactivity, hospitality and security are important to online customers ,C,leading companies began to take the online plunge decades ago ,D,setting up shops in silicon is independent of the cost of computing power Passage 3 An invisible border divides those arguing for computers in the classroom on the behalf of students? career prospects and those arguing for computers in the classroom for broader reasons of radical educational reform. Very few writers on the subject have explored this distinction—indeed, contradiction—which goes to the heart of what is wrong with the campaign to put computers in the classroom. An education that aims at getting a student a certain kind of job is a technical education, justified for reasons radically different from why education is universally required by law. It is not simply to raise everyone?s job prospects that all children are legally required to attend school into their teens. Rather, we have a certain conception of the American citizen, a character who is incomplete if he cannot competently assess how his livelihood and happiness are affected by things outside of himself. But this was not always the case; before it was legally required for all children to attend school until a certain age, it was widely accepted that some were just not equipped by nature to pursue this kind of education. With optimism characteristic of all industrialized countries, we came to accept that everyone is fit to be educated. Computer education advocates forsake this optimistic notion for a pessimism that betrays their otherwise cheery outlook. Banking on the confusion between educational and vocational reasons for bringing computers into schools, computered advocates often emphasize the job prospects of graduates over their educational achievement. There are some good arguments for a technical education given the right kind of student. Many European schools introduce the concept of professional training early on in order to make sure children are properly equipped for the professions they want to join. It is, however, presumptuous to insist that there will only be so many jobs for so many scientists, so many businessmen, so many accountants. Besides, this is unlikely to produce the needed number of every kind of professional in a country as large as ours and where the economy is spread over so many states and involves so many international corporations. But, for a small group of students, professional training might be the way to go since well developed skills, all other factors being equal, can be the difference between having a job and not. Of course, the basics of using any computer these days are very simple. It does not take a lifelong acquaintance to pick up various software programs. If one wanted to become a computer engineer, that is, of course, an entirely different story. Basic computer skills take—at the very longest—a couple of months to learn. In any case, basic computer skills are only complementary to the host of real skills that are necessary to becoming any kind of professional. It should be observed, of course, that no school, vocational or not, is helped by a confusion over its purpose. 19. The author thinks the present rush to put computers in the classroom is. ,A,far-reaching ,B,dubiously oriented ,C,self-contradictory ,D,radically reformatory 20. The belief that education is indispensable to all children. ,A,is indicative of a pessimism in disguise ,B,came into being along with the arrival of computers ,C,is deeply rooted in the minds of compeered advocates ,D,originated from the optimistic attitude of industrialized countries 21. It could be inferred from the passage that in the author?s country the European model of professional training is. ,A,dependent upon the starting age of candidates ,B,worth trying in various social sections ,C,of little practical value ,D,attractive to every kind of professional 22. According to the author, basic computer skills should be. ,A,included as an auxiliary course in school ,B,highlighted in acquisition of professional qualifications ,C,mastered through a lifelong course ,D,equally emphasized by any school, vocational or otherwise Passage 4 When a Scottish research team startled the world by revealing 3 months ago that it had cloned an adult sheep, President Clinton moved swiftly. Declaring that he was opposed to using this unusual animal husbandry technique to clone humans, he ordered that federal funds not be used for such an experiment—although no one had proposed to do so—and asked an independent panel of experts chaired by Princeton President Harold Shapiro to report back to the White House in 90 days with recommendations for a national policy on human cloning. That group—the National Bioethics Advisory Commission (NBAC)—has been working feverishly to put its wisdom on paper, and at a meeting on 17 May, members agreed on a near final draft of their recommendations. possibly that it be made law. But NBAC members are planning to word the recommendation narrowly to avoid new restrictions on research that involves the cloning of human DNA or cells—routine in molecular biology. The panel has not yet reached agreement on a crucial question, however, whether to recommend legislation that would make it a crime for private funding to be used for human cloning. In a draft preface to the recommendations, discussed at the 17 May meeting, Shapiro suggested that the panel had found a broad consensus that it would be “morally unacceptable to attempt to create a human child by adult nuclear cloning.” Shapiro explained during the meeting that the moral doubt stems mainly from fears about the risk to the health of the child. The panel then informally accepted several general conclusions, although some details have not been settled. NBAC plans to call for a continued ban on federal government funding for any attempt to clone body cell nuclei to create a child. Because current federal law already forbids the use of federal funds to create embryos (the earliest stage of human offspring before birth) for research or to knowingly endanger an embryo?s life, NBAC will remain silent on embryo research. NBAC members also indicated that they would appeal to privately funded researchers and clinics not to try to clone humans by body cell nuclear transfer. But they were divided on whether to go further by calling for a federal law that would impose a complete ban on human cloning. Shapiro and most members favored an appeal for such legislation, but in a phone interview, he said this issue was still “up in the air”. 23. We can learn from the first paragraph that. ,A,federal funds have been used in a project to clone humans ,B,the White House responded strongly to the news of cloning ,C,NBAC was authorized to control the misuse of cloning technique ,D,the White House has got the panel?s recommendations on cloning 24. The panel agreed on all of the following except that. ,A,the ban on federal funds for human cloning should be made a law ,B,the cloning of human DNA is not to be put under more control ,C,it is criminal to use private funding for human cloning ,D,it would be against ethical values to clone a human being 25. NBAC will leave the issue of embryo research undiscussed because. ,A,embryo research is just a current development of cloning ,B,the health of the child is not the main concern of embryo research ,C,an embryo?s life will not be endangered in embryo research ,D,the issue is explicitly stated and settled in the law 26. It can be inferred from the last paragraph that. ,A,some NBAC members hesitate to ban human cloning completely ,B,a law banning human cloning is to be passed in no time ,C,privately funded researchers will respond positively to NBAC?s appeal ,D,the issue of human cloning will soon be settled Passage 5 Science, in practice, depends far less on the experiments it prepares than on the preparedness of the minds of the men who watch the experiments. Sir Isaac Newton supposedly discovered gravity through the fall of an apple. Apples had been falling in many places for centuries and thousands of people had seen them fall. But Newton for years had been curious about the cause of the orbital motion of the moon and planets. What kept them in place? Why didn?t they fall out of the sky? The fact that the apple fell down toward the earth and not up into the tree answered the question he had been asking himself about those larger fruits of the heavens, the moon and the planets. How many men would have considered the possibility of an apple falling up into the tree? Newton did because he was not trying to predict anything. He was just wondering. His mind was ready for the unpredictable. Unpredictability is part of the essential nature of research. If you don?t have unpredictable things, you don?t have research. Scientists tend to forget this when writing their cut and dried reports for the technical journals, but history is filled with examples of it. In talking to some scientists, particularly younger ones, you might gather the impression that they find the “scientific method” a substitute for imaginative thought. I?ve attended research conferences where a scientist has been asked what he thinks about the advisability of continuing a certain experiment. The scientist has frowned, looked at the graphs, and said, “the data are still inconclusive.” “We know that,” the men from the budget office have said, “but what do you think? Is it worthwhile going on? What do you think we might expect?” The scientist has been shocked at having even been asked to speculate. What this amounts to, of course, is that the scientist has become the victim of his own writings. He has put forward unquestioned claims so consistently that he not only believes them himself, but has convinced industrial and business management that they are true. If experiments are planned and carried out according to plan as faithfully as the reports in the science journals indicate, then it is perfectly logical for management to expect research to produce results measurable in dollars and cents. It is entirely reasonable for auditors to believe that scientists who know exactly where they are going and how they will get there should not be distracted by the necessity of keeping one eye on the cash register while the other eye is on the microscope. Nor, if regularity and conformity to a standard pattern are as desirable to the scientist as the writing of his papers would appear to reflect, is management to be blamed for discriminating against the “odd balls” among researchers in favor of more conventional thinkers who “work well with the team”. 27. The author wants to prove with the example of Isaac Newton that. ,A,inquiring minds are more important than scientific experiments ,B,science advances when fruitful researches are conducted ,C,scientists seldom forget the essential nature of research ,D,unpredictability weighs less than prediction in scientific research 28. The author asserts that scientists. ,A,shouldn?t replace “scientific method” with imaginative thought ,B,shouldn?t neglect to speculate on unpredictable things ,C,should write more concise reports for technical journals ,D,should be confident about their research findings 29. It seems that some young scientists. ,A,have a keen interest in prediction ,B,often speculate on the future ,C,think highly of creative thinking ,D,stick to “scientific method” 30. The author implies that the results of scientific research. ,A,may not be as profitable as they are expected ,B,can be measured in dollars and cents ,C,rely on conformity to a standard pattern ,D,are mostly underestimated by management Part ? English-Chinese Translation Directions: Read the following passage carefully and then translate the underlined segments into Chinese. Your translation must be written clearly on ANSWER SHEET 2. (15 points) 31)While there are almost as many definitions of history as there are historians, modern practice most closely conforms to one that sees history as the attempt to recreate and explain the significant events of the past. Caught in the web of its own time and place, each generation of historians determines anew what is significant for it in the past. In this search the evidence found is always incomplete and scattered; it is also frequently partial or partisan. The irony of the historian?s craft is that its practitioners always know that their efforts are but contributions to an unending process. 32)Interest in historical methods has arisen less through external challenge to the validity of history as an intellectual discipline and more from internal quarrels among historians themselves. While history once revered its affinity to literature and philosophy, the emerging social sciences seemed to afford greater opportunities for asking new questions and providing rewarding approaches to an understanding of the past. Social science methodologies had to be adapted to a discipline governed by the primacy of historical sources rather than the imperatives of the contemporary world. 33)During this transfer, traditional historical methods were augmented by additional methodologies designed to interpret the new forms of evidence in the historical study. Methodology is a term that remains inherently ambiguous in the historical profession. 34)There is no agreement whether methodology refers to the concepts peculiar to historical work in general or to the research techniques appropriate to the various branches of historical inquiry. Historians, especially those so blinded by their research interests that they have been accused of “tunnel method,” frequently fall victim to the “technical fallacy.” Also common in the natural sciences, the technicist fallacy mistakenly identifies the discipline as a whole with certain parts of its technical implementation. 35)It applies equally to traditional historians who view history as only the external and internal criticism of sources, and to social science historians who equate their activity with specific techniques. Section ? Writing (15 points) 36. Directions: A. Study the following graphs carefully and write an essay in at less than 150 words. B. Your essay must be written clearly on the ANSWER SHEET 2. C. Your essay should cover three points: a. effect of the country?s growing human population on its wildlife, b. possible reasons for the effect, c. your suggestion for wildlife protection Section ? Writing (15 points) 一、审题谋篇 本题命题形式为图表加提纲式写作。首先考生需要就两幅图各自描写,并且更重要的是,得出两幅图之 间的联系,也就是美国人口增加和野生动物灭绝之间的联系,得出的结论应该是美国人口的快速增长是动物 灭绝的原因之一,文章结尾应该落在保护野生动物这一论调上。 由提纲可以看出,文章应该分为三段。第一段应该描写两幅图并得出结论,即,美国人口增加和动物灭 绝之间的联系。段落主题句可以放在段落第一句也可以放在段落末尾处。 第二段主要讨论的问题是人口增长导致野生动物灭绝的原因,这一段需要避免的缺点是就事论事,换句 话说,也就是不能停留在人口增长导致动物物种灭绝这一现象上,而是要讨论这个现象背后的问题,为什么 人口的增长会导致野生动物灭绝。考生需要分析现象背后隐藏的原因,如人类占据了动物生存的空间、破坏 了动物赖以生存的自然环境等等。 第三段考生可就以上问题提出建议,在该段在写作过程中,考生应注意两点:其一,条理清楚,其二, 减少语言错误。 二、参考范文 It is clear from the first graph that the two hundred years from 1800 to 2000 saw a continuous increase in American population. In the second graph we can see that about 70 wild life species had become extinct in a span of three hundred years and the situation was especially serious from 1800 to 1900, with about 50 to 60 species disappearing from the planet. With these two graphs, we can conclude that the growth of population in the United States is closely related to the drop in the number of wild species. In other words, the population explosion has given rise to the mass destructive effect on the wild life. As we all know, the problem of the extinction of wild life results from several factors. In the first place, the ever-increasing population has taken up a large area of space, which used to be the home of many kinds of wild life. Besides, the process of urbanization and industrialization has driven many kinds of wild life away from their shelter to other places where they cannot survive due to the harsh environment. Concerning the extinction of the wild life, some actions should be taken to tackle this problem. First of all, population should be controlled within a reasonable range with effective measures. Moreover, efforts should be made to protect our environment. In a word, it is not easy to find a solution to this frustrating problem. 三、范文点评 文章结构: 该范文从结构上说,首先紧扣提纲,分为三段。第一段采用了先分后总的写作方法,先描写两幅图,进 而做出比较,在段末得出两幅图之间的联系,这也是第一段的主题句。主题句出现在段落末尾还有一个好处,就是可以达到承上启下的作用,为第二段进一步深入论述这一现象的原因做准备。第二段段首承接上段,开段点明主题:这一现象的产生有几个原因。第三段段首为主体句,段落中提出两个建议,段尾进行总结,与 段首形成呼应。 语言亮点: 1. saw:这是在描写在某段时间发生某种变化时,经常使用的一个表达形式,构成为:时间+experience /see /witness +变化+在哪方面。例如:The 25 years of reforming and opening up see a continuous development in China?s economy and society.,改革开放25年来,中国经济和社会都不断发展,。 2. a span of three hundred years:300年的时间内。Span表“延续的一段时间”,一般时间较长。如:life span,寿命,,the whole span of English history,英国历史的全程,。 3. is closely related to:与…密切相关,同义表达为is closely associated with。 4. drop:表示数量的降低,还可以用decline或decrease。 5. give rise to:导致,引起,同义词为cause, bring about, result in。 6. result from:由…引起,urbanization:城市化,industrialization:工业化。 7. concerning:涉及到…,同义词是:about和regarding。 8. tackle:处理,问题等,,同义词有:deal with, resolve, solve。这句话还可以写成:... , some approaches may be adopted in solving this problem。 9. within a reasonable range:在一个合理的范围内。 四、写作误区 篇章结构误区: 1999年的作文中,考生很容易犯两个错误:跑题和论证不深刻。第一个错误主要是因为考生审题不清所致,有些考生看到了“The Ups and Downs of Population Growth”就误认为这是一篇关于人口增长与控制的作文,最后甚至联想到了计划生育政策,这都是审题不清的结果。考生犯的第二个比较明显的错误就是论证深度不够,这个问题尤其在第二段较为明显。文章第二段主要要求“论证这一结果的原因”,这一结果指的是第一段中分析的人口增长与野生动物灭绝之间的联系,那么考生分析的重点应该是为什么人口的不断增加会导致野生动物的持续减少。结果有些考生根本没有考虑到人口增加会占用野生动物的生存空间、破坏环境等,而是一味地反复说“人口增长使得野生动物的数量减少”。这一现象也反映出考生平时在知识和思考的广度和深度上都有待进一步提高。 语言表达错误: ?谓语动词错误: Concerning myself, I think that people should paid much attention to the protection of wildlife.,As far as I am concerned, I think that people should pay much attention to the protection of wildlife., ?近形词混淆: The increasing population effects on its wildlife.,The increasing population affects / have effects on its wildlife., ?词的数用错: So some of the wild lives become homeless and extinction.,So some of the wildlife become homeless and extinct., ?不间断句子: More and more people came to live some wild animals place, these animals moved to other places.,More and more people came to occupy some wild animals home, so these animals had to move to other places., ?不知所云: This measure should be a protection of education.,By this way, we can educate the public about the protection of wild animals., ?时态语态不一致: If the law said that the man who destroy the wildlife?s environment will be fined 10,000 dollars, there will be fewer people to do that.,If the law says that the man who destroys the wildlife?s environment will be fined 10,000 dollars, there will be fewer offenders., ?中式英语: With the increase of population, the grain needed by people grow too.,With the increase of population, peoples need for grain also grows / increases., ?从句结构不完整: It is the reason that the number of wild animal?s decreasing.,It is the reason why the number of wild animals is decreasing., ?综合性错误: The first step is that take the thought of environment up in everyone?s heart.,The first step is to raise the public?s consciousness of environment.) 2000年全真试题 Part ?Close Test Directions: For each numbered blank in the following passage, there are four choices marked ,A,, ,B,, ,C, and ,D,. Choose the best one and mark your answer on ANSWER SHEET 1 by blackening the corresponding letter in the brackets with a pencil. (10 points) ?If a farmer wishes to succeed, he must try to keep a wide gap between his consumption and his production. ?He must store a large quantity of grain 1 consuming all his grain immediately. ?He can continue to support himself and his family 2 he produces a surplus. ?He must use this surplus in three ways: as seed for sowing, as an insurance 3 the unpredictable effects of bad weather and as a commodity which he must sell in order to 4 old agricultural implements and obtain chemical fertilizers to 5 the soil. ?He may also need money to construct irrigation 6 and improve his farm in other ways. ?If no surplus is available, a farmer cannot be 7 . ?He must either sell some of his property or 8 extra funds in the form of loans. ?Naturally he will try to borrow money at a low 9 of interest, but loans of this kind are not 10 obtainable. ,139 words, 1.,A,other than ,B,as well as ,C,instead of ,D,more than 2.,A,only if ,B,much as ,C,long before ,D,ever since 3.,A,for ,B,against ,C,of ,D,towards 4.,A,replace ,B,purchase ,C,supplement ,D,dispose 5.,A,enhance ,B,mix ,C,feed ,D,raise 6.,A,vessels ,B,routes ,C,paths ,D,channels 7.,A,self-confident ,B,self-sufficient ,C,self-satisfied ,D,self-restrained 8.,A,search ,B,save ,C,offer ,D,seek 9.,A,proportion ,B,percentage ,C,rate ,D,ratio 10.,A,genuinely ,B,obviously ,C,presumably ,D,frequently Part ?Reading Comprehension Directions: Each of the passages below is followed by some questions. For each question there are four answers marked,A,, ,B,,,C,and,D,. Read the passages carefully and choose the best answer to each of the questions. Then mark your answer on ANSWER SHEET 1 by blackening the corresponding letter in the brackets with a pencil. (40 points) Passage 1 ?A history of long and effortless success can be a dreadful handicap, but, if properly handled, it may become a driving force. ?When the United States entered just such a glowing period after the end of the Second World War, it had a market eight times larger than any competitor, giving its industries unparalleled economies of scale. ?Its (11)scientists were the worlds best; its workers the most skilled. ?America and Americans were prosperous beyond the dreams of the Europeans and Asians whose economies the war had destroyed. ?It was inevitable that this primacy should have narrowed as other countries grew richer. ?Just as inevitably, the retreat from predominance proved painful. ?By the mid-1980s Americans had found themselves at a loss over their fading industrial competitiveness. ?Some huge American industries, such as consumer electronics, had shrunk or vanished in the face of foreign competition. ?By 1987 there was only one American television maker left, Zenith. ()12Foreign-made cars and ?(Now there is none: Zenith was bought by South Korea?s LG Electronics in July.) ? textiles were sweeping into the domestic market. -tool industry was on the ropes. ?For a while it looked as though the making of semiconductors, which America had invented and which sat at the heart of the new computer age, was going to be the next casualty. ?All of this caused a crisis of confidence. ?Americans stopped taking prosperity for granted. ?They began to believe that their way of doing business was failing, and that their incomes would therefore shortly begin to fall as well. ?The mid-1980s brought one inquiry after another into the causes of America?s industrial decline. ?Their sometimes sensational findings were filled with warnings about the growing competition from overseas. ?How things have changed! ?In 1995 the United States can look back on five years of solid growth while ()14Japan has been struggling. ?Few Americans attribute this solely to such obvious causes as a devalued dollar or the turning of the business cycle. ?Self-doubt has yielded to blind pride. ?“American industry has changed its structure, has gone on a diet, has learnt to be more quick-witted,” according to Richard Cavanaugh, executive dean of Harvard?s Kennedy School of Government. ?“It makes me proud to be an American just to see how our businesses are improving their productivity,” says Stephen Moore of the Cato Institute, a think-tank in Washington, DC. ?And William Shaman of the Harvard Business School believes that people will look back on this period as “a golden age of business management in the United States.”,429 words, 11. The U.S. achieved its predominance after World War II because. ,A,it had made painstaking efforts towards this goal ,B,its domestic market was eight times larger than before ,C,the war had destroyed the economies of most potential competitors ,D,the unparalleled size of its workforce had given an impetus to its economy 12. The loss of U.S. predominance in the world economy in the 1980s is manifested in the fact that the American. ,A,TV industry had withdrawn to its domestic market ,B,semiconductor industry had been taken over by foreign enterprises ,C,machine-tool industry had collapsed after suicidal actions ,D,auto industry had lost part of its domestic market 13. What can be inferred from the passage? ,A,It is human nature to shift between self-doubt and blind pride. ,B,Intense competition may contribute to economic progress. ,C,The revival of the economy depends on international cooperation. ,D,A long history of success may pave the way for further development. 14. The author seems to believe the revival of the U.S. economy in the 1990s can be attributed to the. ,A,turning of the business cycle ,B,restructuring of industry ,C,improved business management ,D,success in education Passage 2 ()15?Being a man has always been dangerous. ?There are about 105 males born for every 100 females, but this ratio drops to near balance at the age of maturity, and among 70-year-olds there are twice as many women as men. ?But the great universal of male mortality is being changed. ?Now, boy babies survive almost as well as girls do. ?This means that, for the first time, there will be an excess of boys in those crucial years when they are searching for a mate. ?More important, another chance for natural selection has been removed. ?Fifty years ago, the chance of a certain death. ?Today it makes almost no difference. Since much of the variation is due to genes, one more agent of evolution has gone. ?There is another way to commit evolutionary suicide: stay alive, but have fewer children. ?Few people are as fertile as in the past. ?Except in some religious communities, very few women have 15 children. ?Nowadays the number of births, like the age of death, has become average. ?Most of us have roughly the same number of offspring. ()16Again, differences between people and the opportunity for natural selection to take advantage of it have ? diminished. ?India shows what is happening. for the remaining tribal peoples. ?The grand mediocrity of today—everyone being the same in survival and number of offspring—means that natural selection has lost 80% of its power in upper-middle-class India compared to the tribes. For us, this means that evolution is over; the biological Utopia has arrived. ?Strangely, it has involved little physical change. ?No other species fills so many places in nature. ?But in the past 100, 000 years—even the past ()17100 years—our lives have been transformed but our bodies have not. ?We did not evolve, because machines and society did it for us. ?Darwin had a phrase to describe those ignorant of evolution: they “look at an organic being as a savage looks at a ship, as at something wholly beyond his comprehension.” No doubt we will remember a 20th ur descendants may be at how far from Utopia we were, they will look just like us.,406 words, 15. What used to be the danger in being a man according to the first paragraph? ,A,A lack of mates. ,B,A fierce competition. ,C,A lower survival rate. ,D,A defective gene. 16. What does the example of India illustrate? ,A,Wealthy people tend to have fewer children than poor people. ,B,Natural selection hardly works among the rich and the poor. ,C,The middle class population is 80% smaller than that of the tribes. ,D,India is one of the countries with a very high birth rate. 17. The author argues that our bodies have stopped evolving because. ,A,life has been improved by technological advance ,B,the number of female babies has been declining ,C,our species has reached the highest stage of evolution ,D,the difference between wealth and poverty is disappearing 18. Which of the following would be the best title for the passage? ,A,Sex Ratio Changes in Human Evolution. ,B,Ways of Continuing Man?s Evolution. ,C,The Evolutionary Future of Nature. ,D,Human Evolution Going Nowhere. Passage 3 ()20?When a new movement in art attains a certain fashion, it is advisable to find out what its advocates are aiming at, for, however farfetched and unreasonable their principles may seem today, it is possible that in years to come they may be regarded as normal. ?With regard to Futurist poetry, however, the case is rather difficult, for whatever Futurist poetry may be—even admitting that the theory on which it is based may be right—it can hardly be classed as Literature. ?This, in brief, is what the Futurist says: for a century, past conditions of life have been conditionally speeding up, till now we live in a world of noise and violence and speed. ?Consequently, our feelings, thoughts and emotions ()21have undergone a corresponding change. ?This speeding up of life, says the Futurist, requires a new form of expression. ?We must speed up our literature too, if we want to interpret modern stress. ?We must pour out a large stream of essential words, unhampered by stops, or qualifying adjectives, or finite verbs. ?Instead of describing sounds we must make up words that imitate them; we must use many sizes of type and different colored inks on the same page, and shorten or lengthen words at will. ?Certainly their descriptions of battles are confused. ?But it is a little upsetting to read in the explanatory notes that a certain line describes a fight between a Turkish and a Bulgarian officer on a bridge off which they both fall into the river —and then to find that the line consists of the noise of their falling and the weights of the officers: “Pluff! Pluff! A hundred and eighty-five kilograms.” ()22?This, though it fulfills the laws and requirements of Futurist poetry, can hardly be classed as Literature. ?All the same, no thinking man can refuse to accept their first proposition: that a great change in our emotional life calls for a change of expression. ?The whole question is really this: have we essentially changed?,334 words, 19. This passage is mainly. ,A,a survey of new approaches to art ,B,a review of Futurist poetry ,C,about merits of the Futurist movement ,D,about laws and requirements of literature 20. When a novel literary idea appears, people should try to. ,A,determine its purposes ,B,ignore its flaws ,C,follow the new fashions ,D,accept the principles 21. Futurists claim that we must. ,A,increase the production of literature ,B,use poetry to relieve modern stress ,C,develop new modes of expression ,D,avoid using adjectives and verbs 22. The author believes that Futurist poetry is. ,A,based on reasonable principles ,B,new and acceptable to ordinary people ,C,indicative of a basic change in human nature ,D,more of a transient phenomenon than literature Passage 4 ()23?Aimlessness has hardly been typical of the postwar Japan whose productivity and social harmony are the envy of the United States and Europe. ?But increasingly the Japanese are seeing a decline of the traditional work-moral values. ?Ten years ago young people were hardworking and saw their jobs as their primary reason for being, but now Japan has largely fulfilled its economic needs, and young people don?t know where they should go next. ?The coming of age of the postwar baby boom and an entry of women into the male-dominated job market have limited the opportunities of teen-agers who are already questioning the heavy personal sacrifices involved in climbing Japan?s rigid social ladder to good schools and jobs. ?In a recent survey, it was found that only 24.5 percent of Japanese students were fully satisfied with school life, compared with 67.2 percent of students in the United States. ?In addition, far more Japanese workers expressed dissatisfaction with their jobs than did their counterparts in the 10 other countries surveyed. ?While often praised by foreigners for its emphasis on the basics, Japanese education tends to stress test taking ()25and mechanical learning over creativity and self-expression. ?“Those things that do not show up in the test scores—personality, ability, courage or humanity—are completely ignored,” says Toshiki Kaifu, chairman of the ruling Liberal Democratic Party?s education committee. ?“Frustration against this kind of thing leads kids to drop out and run wild.” ?Last year Japan experienced 2,125 incidents of school violence, including 929 assaults on teachers. ?Amid the outcry, many conservative leaders are seeking a return to the prewar emphasis on moral education. ?Last year Mitsuo Setoyama, who was then education minister, raised eyebrows when he argued that liberal reforms introduced by the American occupation authorities after World War II had weakened the “Japanese morality of respect for parents.” ()26?But that may have more to do with Japanese life-styles. ?“In Japan,” says educator Yoko Muro, “it?s never a question of whether you enjoy your job and your life, but only how much you can endure.” ?With economic growth has come centralization; fully 76 percent of Japan?s 119 million citizens live in cities where community and the extended family have been abandoned in favor of isolated, two-generation households. ?Urban Japanese have long endured lengthy commutes (travels to and from work) and crowded living conditions, but as the old group and family values weaken, the discomfort is beginning to tell. ?In the past decade, the Japanese divorce rate, while still well below that of the United States, has increased by more than 50 percent, and suicides have increased by nearly one-quarter.,447 words, 23. In the Westerners? eyes, the postwar Japan was. ,A,under aimless development,B,a positive example ,C,arrival to the West,D,on the decline 24. According to the author, what may chiefly be responsible for the moral decline of Japanese society? ,A,Women?s participation in social activities is limited. ,B,More workers are dissatisfied with their jobs. ,C,Excessive emphasis has been placed on the basics. ,D,The life-style has been influenced by Western values. 25. Which of the following is true according to the author? ,A,Japanese education is praised for helping the young climb the social ladder. ,B,Japanese education is characterized by mechanical learning as well as creativity. ,C,More stress should be placed on the cultivation of creativity. ,D,Dropping out leads to frustration against test taking. 26. The change in Japanese life-style is revealed in the fact that. ,A,the young are less tolerant of discomforts in life ,B,the divorce rate in Japan exceeds that in the U.S. ,C,the Japanese endure more than ever before ,D,the Japanese appreciate their present life Passage 5 ()27?If ambition is to be well regarded, the rewards of ambition—wealth, distinction, control over one?s destiny—must be deemed worthy of the sacrifices made on ambition?s behalf. ?If the tradition of ambition is to have vitality, it must be widely shared; and it especially must be highly regarded by people who are themselves admired, ()28the educated not least among them. ?In an odd way, however, it is the educated who have claimed to have given up on ambition as an ideal. ?What is odd is that they have perhaps most benefited from ambition—if not always their own then that of their parents and grandparents. ?There is a heavy note of hypocrisy in this, a case of closing the barn door after the horses have escaped—with the educated themselves riding on them. ?Certainly people do not seem less interested in success and its signs now than formerly. ?Summer homes, European travel, BMWs—the locations, place names and name brands may change, but such items do not seem less ()29in demand today than a decade or two years ago.?What has happened is that people cannot confess fully to their dreams, as easily and openly as once they could, lest they be thought pushing, acquisitive and vulgar. ?Instead, we are treated to fine hypocritical spectacles, which now more than ever seem in ample supply: the critic of American materialism with a Southampton summer home; the publisher of radical books who takes his meals in three-star restaurants; the journalist advocating participatory democracy in all phases of life, whose own children are enrolled in private schools. ?For such people and many more perhaps not so exceptional, the proper formulation is, “Succeed at all costs but avoid appearing ambitious.” ?The attacks on ambition are many and come from various angles; its public defenders are few and unimpressive, where they are not extremely unattractive. ?As a result, the support for ambition as a healthy impulse, a quality to be admired and fixed in the mind of the young, is probably lower than it has ever been in the United States. ?This does not mean that ambition is at an end, that people no longer feel its stirrings and promptings, but only that, no longer openly honored, it is less openly professed. ?Consequences follow from this, of course, some of which are that ambition is driven underground, or made sly. ?Such, then, is the way things stand: on the left angry critics, on the right stupid supporters, and in the middle, as usual, the majority of earnest people trying to get on in life. ,431 words, 27. It is generally believed that ambition may be well regarded if. ,A,its returns well compensate for the sacrifices ,B,it is rewarded with money, fame and power ,C,its goals are spiritual rather than material ,D,it is shared by the rich and the famous 28. The last sentence of the first paragraph most probably implies that it is. ,A,customary of the educated to discard ambition in words ,B,too late to check ambition once it has been let out ,C,dishonest to deny ambition after the fulfillment of the goal ,D,impractical for the educated to enjoy benefits from ambition 29. Some people do not openly admit they have ambition because. ,A,they think of it as immoral ,B,their pursuits are not fame or wealth ,C,ambition is not closely related to material benefits ,D,they do not want to appear greedy and contemptible 30. From the last paragraph the conclusion can be drawn that ambition should be maintained. ,A,secretly and vigorously ,B,openly and enthusiastically ,C,easily and momentarily ,D,verbally and spiritually Part ?English-Chinese Translation Directions: Read the following passage carefully and then translate the underlined segments into Chinese. Your translation must be written neatly on ANSWER SHEET 2. (15 points) Governments throughout the world act on the assumption that the welfare of their people depends largely on the economic strength and wealth of the community. 31)Under modern conditions, this requires varying measures of centralized control and hence the help of specialized scientists such as economists and operational research experts. 32)Furthermore, it is obvious that the strength of a country?s economy is directly bound up with the efficiency of its agriculture and industry, and that this in turn rests upon the efforts of scientists and technologists of all kinds. It also means that governments are increasingly compelled to interfere in these sectors in order to step up production and ensure that it is utilized to the best advantage. For example, they may encourage research in various ways, including the setting up of their own research centers; they may alter the structure of education, or interfere in order to reduce the wastage of natural resources or tap resources hitherto unexploited; or they may cooperate directly in the growing number of international projects related to science, economics and industry. In any case, all such interventions are heavily dependent on scientific advice and also scientific and technological manpower of all kinds. 33)Owing to the remarkable development in mass-communications, people everywhere are feeling new wants and are being exposed to new customs and ideas, while governments are often forced to introduce still further innovations for the reasons given above. At the same time, the normal rate of social change throughout the world is taking place at a vastly accelerated speed compared with the past. For example, 34)in the early industrialized countries of Europe the process of industrialization—with all the far-reaching changes in social patterns that followed—was spread over nearly a century, whereas nowadays a developing nation may undergo the same process in a decade or so. All this has the effect of building up unusual pressures and tensions within the community and consequently presents serious problems for the governments concerned. 35)Additional social stresses may also occur because of the population explosion or problems arising from mass migration movements—themselves made relatively easy nowadays by modern means of transport. As a result of all these factors, governments are becoming increasingly dependent on biologists and social scientists for planning the appropriate programs and putting them into effect. ,390 words, Section ? Writing (15 points) Directions: A. Study the following two pictures carefully and write an essay of at least 150 words. B. Your essay must be written neatly on ANSWER SHEET 2. C. Your essay should meet the requirements below: 1)Describe the pictures. 2)Deduce the purpose of the painter of the pictures. 3)Suggest counter-measures. Section ? Writing (15 points) 一、审题谋篇 本题命题形式是图画式写作。该图片由上下两部分组成,展现的是对比关系。图片的主体是“船和鱼”, 上部分的图片中,船有一艘,鱼则很多,而下部分图片则完全相反,船变得特别多,鱼却只有一条。图片上 的“1900”和“1995”表明两部分展现的是不同时间段的不同景象。图片下面的标题“A history of world commercial fishing”揭示了图片所反映的现象的背景和原因。 分析图片后,接下来进行构思。根据题目要求,文章可以分为三部分写。第一部分对图片进行描写:因 为两幅图形成的是一种鲜明的对比,描写时可采用对比的句式,注意表对比、转折的连接词的选用。第二部 分分析画者的意图:结合图片及标题可知画者在揭示大肆捕捞使得海洋鱼类资源迅速减少的现象。进一步剖 析,则可得出图片传达的深层含义是批判人类对自然资源的滥用,如果透过现象,分析原因,还可以得出: 这一切破坏的根源是商业利益的驱使。第三部分提出相应的解决 方案 气瓶 现场处置方案 .pdf气瓶 现场处置方案 .doc见习基地管理方案.doc关于群访事件的化解方案建筑工地扬尘治理专项方案下载 ,需要列出具体可行的措施和建议。具 体建议可以从负责部门和普通个人的角度分别提出。需要注意的是本文字数限制为150字左右,因此观点表达 应力求简单明了。 二、参考范文 A big contrast is shown in the picture. In 1900 there was only one ship in the sea rich in fish, while in 1995, there became only one fish in the sea crowded with ships. The caption demonstrates that it was the ever-developing fishing industry that caused the devastating effects on the fishing resource. The picture conveys a deep concern about the ecological disaster which is being brought about by the over-fishing driven by short-term commercial interests. It also implies a more worrisome consequence. The profit-oriented behavior may penetrate into other fields, which results in the abuse of more resources. To solve this concern, we should spare no efforts in protecting natural resources. Laws should be enforced to prohibit the actions of blind exploitation, such as over-fishing in this picture. In addition, people should be educated to realize the value of these resources and voluntarily protect them. 三、范文点评 文章结构: 文章以描写图片开篇,自然过渡到图片含义的阐释和引申,并在充分论证的基础上提出合理的建议。整 篇文章自然连贯,层次清晰。 语言亮点: 1. 平行结构表示对比,增强语势,同义词的运用,避免重复 In 1900 there was only one ship in the sea rich in fish, while in 1995, there became only one fish in the sea crowded with ships. 2. 强调句突显文章主题 It was the ever-developing fishing industry that... 3. 非限制性定语从句使语义紧凑、层次分明 The profit-oriented behavior may penetrate into other fields, which ... 4. 合成形容词 Ever-developing: ever+-ing表示“一直,不断……”,该词的意思是“不断发展”。又如:ever-increasing,不断增长, Profit-oriented: 名词+oriented, 意思是“以…… 为中心”,该词的意思是“以利益为中心”。又如:exam-oriented (以考试为中心) Over-fishing:over+-ing/ +ed 表示“过度的……”,该词的意思是“过度捕捞”。又如over-cooked(煮过了的) 5.词汇亮点 devastating:毁坏性的。一般修饰impact, effect, consequence等,如:Acid rain has a devastating effect on the forest. 酸雨对森林有很大的破坏性 convey:表达,传达。如:I want to convey to children that reading is one of life?s happiest things. 我想告诉孩子,在人的一生中,阅读是一件最快乐的事情 imply:暗示,意味。如:I don?t imply that you are wrong. 我没有暗示你错了 penetrate:渗透到,深入到。如:Explorers penetrated deep into unknown regions. 探险家深入到未知地域。 spare no efforts: “不遗余力”。类似表达有:work very hard, put in great effort, use all one?s power prohibit:禁止,阻止。搭配形式有prohibit sb from doing sth,如:They are prohibited from revealing details about the candidates. 严禁他们泄露候选人的具体信息 四、写作误区 篇章结构误区: 考生在主题把握上出现偏差,其原因如下:一,受惯有思维的影响。一些考生看到图片表现了鱼类资源减少,就想当然地把这一现象与pollution联系到了一起,从而大谈环境污染的坏处以及控制的措施。二,审题不全面。有些考生只看了图片的文字,通篇只写捕鱼业的历史。或者忽视图片文字,只谈渔类资源减少,避而不谈捕鱼业在其中的影响。三,立意狭隘。部分考生停留在现象的表面,对图片仅进行了初步分析,而对现象的来龙去脉以及其中蕴含的深刻的因果关系没有进行剖析。比如有些考生只得出“船多了鱼就少了”这种表面的联系,从而仅仅提出限制渔船数量的建议。 语言表达错误: 1. 用词不当 Then people began to fish ,catch,plenty of fishes in the ocean because they had bigger boats. Environmental education to children is one way(solution) to the problem of sea pollution. 2. 中式英语 In 1995, the fishing ships were many.,In 1995, there were many fishing ships., Ships were many more than in 1900.,There were many more ships in 1995 than in 1900., 3. 文体不统一 Ok. That?s what I want to say. Do you agree with me?,I hope my article will help more and more people realize the importance of protecting natural resources., 4. 综合错误 The decrease of the number of fish in the sea is not a simple problem which has many reasons.,The decrease of the number of fish, which is not a simple problem, has many reasons., 五、相关链接 语法回顾: 1. 强调句 “It is (was) +被强调的部分+that+剩余部分”。如:It is only when one is ill that one realize the value of health. It is this novel that they talked about last week. 2. 非限制性定语从句 修饰整个主句,使句意层次更清晰。如:She is always helping me, which touches me a lot. 表达补充: 1. 表对比转折关系的连接词 but, however, nevertheless, yet, while, on the contrary, in contrast with (to) 2. 表列举的过渡词 ?for one thing,...for another... st... second... finally ?in the first place... in the second place first of all... next ?to begin with... what?s more addition, besides, also, on top of that 3. 环境话题 ?environmental degradation /be degraded,环境退化, ?The extinction rate of species ,物种灭绝, collapse of fish stocks,鱼类储量锐减, ?ecological state of the world,世界生态环境的状态, ?threaten,v.,/threats to,威胁, used unsustainably( 非可持续性利用) ?pose serious risks for,构成严重危害, ... . has the potential to greatly enlarge the areas in which... . is a threat, ……极有可能扩大……所产生的威 胁, ?The range of... . is also likely to increase/decline,……的范围也有可能扩大、缩小, With the increases/decline in... , ...be also predicted,随着……,将会出现……, ...is dramatic, great, surprising, shocking, excessive(过度,极端的) 2001年全真试题 Part I Cloze Test Directions: For each numbered blank in the following passage, there are four choices marked [A], [B], [C] and [D]. Choose the best one and mark your answer on ANSWER SHEET 1 by blackening the corresponding letter in the brackets with a pencil. (10 points) The government is to ban payments to witnesses by newspapers seeking to buy up people involved in prominent cases 1 the trial of Rosemary West. In a significant 2 of legal controls over the press, Lord Irvine, the Lord Chancellor, will introduce a 3 bill that will propose making payments to witnesses 4 and will strictly control the amount of 5 that can be given to a case 6 a trial begins. In a letter to Gerald Kaufman, chairman of the House of Commons media select committee, Lord Irvine said he 7 with a committee report this year which said that self regulation did not 8 sufficient control. 9 of the letter came two days after Lord Irvine caused a 10 of media protest when he said the 11 of privacy controls contained in European legislation would be left to judges 12 to Parliament. The Lord Chancellor said introduction of the Human Rights Bill, which 13 the European Convention on Human Rights legally 14 in Britain, laid down that everybody was 15 to privacy and that public figures could go to court to protect themselves and their families. “Press freedoms will be in safe hands 16 our British judges,” he said. Witness payments became an 17 after West was sentenced to 10 life sentences in 1995. Up to 19 witnesses were 18 to have received payments for telling their stories to newspapers. Concerns were raised 19 witnesses might be encouraged exaggerate their stories in court to 20 guilty verdicts. 1.,A,as to ,B,for instance ,C,in particular ,D,such as 2.,A,tightening ,B,intensifying ,C,focusing ,D,fastening 3.,A,sketch ,B,rough ,C,preliminary ,D,draft 4.,A,illogical ,B,illegal ,C,improbable ,D,improper 5.,A,publicity ,B,penalty ,C,popularity ,D,peculiarity 6.,A,since ,B,if ,C,before ,D,as 7.,A,sided ,B,shared ,C,complied ,D,agreed 8.,A,present ,B,offer ,C,manifest ,D,indicate 9.,A,Release ,B,Publication ,C,Printing ,D,Exposure 10.,A,storm ,B,rage ,C,flare ,D,flash 11.,A,translation ,B,interpretation ,C,exhibition ,D,demonstration 12.,A,better than ,B,other than ,C,rather than ,D,sooner than 13.,A,changes ,B,makes ,C,sets ,D,turns 14.,A,binding ,B,convincing ,C,restraining ,D,sustaining 15.,A,authorized ,B,credited ,C,entitled ,D,qualified 16.,A,with ,B,to ,C,from ,D,by 17.,A,impact ,B,incident ,C,inference ,D,issue 18.,A,stated ,B,remarked ,C,said ,D,told 19.,A,what ,B,when ,C,which ,D,that 20.,A,assure ,B,confide ,C,ensure ,D,guarantee Part II Reading Comprehension Directions: Each of the passages below is followed by some questions. For each questions there are four answers marked [A], [B], [C] and [D]. Read the passages carefully and choose the best answer to each of the questions. Then mark your answer on ANSWER SHEET 1 by blackening the corresponding letter in the brackets with a pencil. (40 points) Passage 1 Specialisation can be seen as a response to the problem of an increasing accumulation of scientific knowledge. By splitting up the subject matter into smaller units,one man could continue to handle the information and use it as the basis for further research. But specialisation was only one of a series of related developments in science affecting the process of communication. Another was the growing professionalisation of scientific activity. No clear-cut distinction can be drawn between professionals and amateurs in science: exceptions can be found to any rule. Nevertheless, the word “amateur” does carry a connotation that the person concerned is not fully integrated into the scientific community and, in particular, may not fully share its values. The growth of specialisation in the nineteenth century, with its consequent requirement of a longer, more complex training, implied greater problems for amateur participation in science. The trend was naturally most obvious in those areas of science based especially on a mathematical or laboratory training, and can be illustrated in terms of the development of geology in the United Kingdom. A comparison of British geological publications over the last century and a half reveals not simply an increasing emphasis on the primacy of research, but also a changing definition of what constitutes an acceptable research paper. Thus, in the nineteenth century, local geological studies represented worthwhile research in their own right; but, in the twentieth century, local studies have increasingly become acceptable to professionals only if they incorporate, and reflect on, the wider geological picture. Amateurs, on the other hand, have continued to pursue local studies in the old way. The overall result has been to make entrance to professional geological journals harder for amateurs, a result that has been reinforced by the widespread introduction of refereeing, first by national journals in the nineteenth century and then by several local geological journals in the twentieth century. As a logical consequence of this development, separate journals have now appeared aimed mainly towards either professional or amateur readership. A rather similar process of differentiation has led to professional geologists coming together nationally within one or two specific societies, whereas the amateurs have tended either to remain in local societies or to come together nationally in a different way. Although the process of professionalisation and specialisation was already well under way in British geology during the nineteenth century, its full consequences were thus delayed until the twentieth century. In science generally, however, the nineteenth century must be reckoned as the crucial period for this change in the structure of science. 21. The growth of specialisation in the 19th century might be more clearly seen in sciences such as _______. [A]sociology and chemistry [B]physics and psychology [C]sociology and psychology [D]physics and chemistry 22. We can infer from the passage that _______. [A] there is little distinction between specialisation and professionalisation [B] amateurs can compete with professionals in some areas of science [C] professionals tend to welcome amateurs into the scientific community [D] amateurs have national academic societies but no local ones 23. The author writes of the development of geology to demonstrate ______. [A] the process of specialisation and professionalisation [B] the hardship of amateurs in scientific study [C] the change of policies in scientific publications [D] the discrimination of professionals against amateurs 24. The direct reason for specialisation is _______. [A] the development in communication [B] the growth of professionalisation [C] the expansion of scientific knowledge [D] the splitting up of academic societies Passage 2 A great deal of attention is being paid today to the so-called digital divide-the division of the world into the info (information) rich and the info poor. And that divide does exist today. My wife and I lectured about this looming danger twenty years ago. What was less visible then, however, were the new, positive forces that work against the digital divide. There are reasons to be optimistic. There are technological reasons to hope the digital divide will narrow. As the Internet becomes more and more commercialized, it is in the interest of business to universalize access-after all, the more people online, the more potential customers there are. More and more governments, afraid their countries will be left behind, want to spread Internet access. Within the next decade or two, one to two billion people on the planet will he netted together. As a result, I now believe the digital divide will narrow rather than widen in the years ahead. And that is very good news because the Internet may well be the most powerful tool for combating world poverty that we?ve ever had. Of course, the use of the Internet isn?t the only way to defeat poverty. And the Internet is not the only tool we have. But it has enormous potential. To take advantage of this tool, some impoverished countries will have to get over their outdated anti-colonial prejudices with respect to foreign investment. Countries that still think foreign investment is an invasion of their sovereignty might well study the history of infrastructure(the basic structural foundations of a society)in the United States. When the United States built its industrial infrastructure, it didn?t have the capital to do so. And that is why America?s Second Wave infrastructure-including roads, harbors, highways, ports and so on-were built with foreign investment. The English, the Germans, the Dutch and the French were investing in Britain?s former colony. They financed them. Immigrant Americans built them. Guess who owns them now? The Americans. I believe the same thing would be true in places like Brazil or anywhere else for that matter. The more foreign capital you have helping you build your Third Wave infrastructure, which today is an electronic infrastructure, the better off you?re going to be. That doesn't mean lying down and becoming fooled, or letting foreign corporations run uncontrolled. But it does mean recognizing how important they can be in building the energy and telecom infrastructures needed to take full advantage of the Internet. 25. Digital divide is something _______. [A] getting worse because of the Internet [B] the rich countries are responsible for [C] the world must guard against [D] considered positive today 26. Governments attach importance to the Internet because it _______. [A] offers economic potentials [B] can bring foreign funds [C] can soon wipe out world poverty [D] connects people all over the world 27. The writer mentioned the case of the United States to justify the policy of _______. [A] providing financial support overseas [B] preventing foreign capital?s control [C] building industrial infrastructure [D] accepting foreign investment 28. It seems that now a country?s economy depends much on ______. [A] how well-developed it is electronically [B] whether it is prejudiced against immigrants [C] whether it adopts America?s industrial pattern [D] how much control it has over foreign corporations Passage 3 Why do so many Americans distrust what they read in their newspapers? The American Society of Newspaper Editors is trying to answer this painful question. The organization is deep into a long self-analysis known as the journalism credibility project. Sad to say, this project has turned out to be mostly low-level findings about factual errors and spelling and grammar mistakes, combined with lots of headscratching puzzlement about what in the world those readers really want. But the sources of distrust go way deeper. Most journalists learn to see the world through a set of standard templates (patterns) into which they plug each day?s events. In other words, there is a conventional story line in the newsroom culture that provides a backbone and a ready-made narrative structure for otherwise confusions news. There exists a social and cultural disconnect between journalists and their readers which helps explain why the “standard templates” of the newsroom seem alien many readers. In a recent survey, questionnaires were sent to reporters in five middle size cities around the country, plus one large metropolitan area. Then residents in these communities were phoned at random and asked the same questions. Replies show that compared with other Americans, journalists are more likely to live in upscale neighborhoods, have maids, own Mercedeses, and trade stocks, and they?re less likely to go to church, do volunteer work, or put down roots in community. Reporters tend to be part of a broadly defined social and cultural elite, so their work tends to reflect the conventional values of this elite. The astonishing distrust of the news media isn?t rooted in inaccuracy or poor reportorial skills but in the daily clash of world views between reporters and their readers. This is an explosive situation for any industry, particularly a declining one. Here is a troubled business that keeps hiring employees whose attitudes vastly annoy the customers. Then it sponsors lots of symposiums and a credibility project dedicated to wondering why customers are annoyed and fleeing in large numbers. But it never seems to get around to noticing the cultural and class biases that so many former buyers are complaining about. If it did, it would open up its diversity program, now focused narrowly on race and gender, and look for reporters who differ broadly by outlook, values, education, and class. 29. What is the passage mainly about? [A] needs of the readers all over the world. [B] causes of the public disappointment about newspapers. [C] origins of the declining newspaper industry. [D] aims of a journalism credibility project. 30. The results of the journalism credibility project turned out to be ______. [A] quite trustworthy [B] somewhat contradictory [C] very illuminating [D] rather superficial 31. The basic problem of journalists as pointed out by the writer lies in their ______. [A] working attitude [B] conventional lifestyle [C] world outlook [D]educational background 32. Despite its efforts, the newspaper industry still cannot satisfy the readers owing to its_______. [A] failure to realize its real problem [B] tendency to hire annoying reporters [C] likeliness to do inaccurate reporting [D] prejudice in matters of race and gender Passage 4 The world is going through the biggest wave of mergers and acquisitions ever witnessed. The process sweeps from hyperactive America to Europe and reaches the emerging countries with unsurpassed might. Many in these countries are looking at this process and worrying: "Won't the wave of business concentration turn into an uncontrollable anti-competitive force?" There's no question that the big are getting bigger and more powerful. Multinational corporations accounted for less than 20% of international trade in 1982. Today the figure is more than 25% and growing rapidly. International affiliates account for a fast-growing segment of production in economies that open up and welcome foreign investment. In Argentina, for instance, after the reforms of the early 1990s, multinationals went from 43% to almost 70% of the industrial production of the 200 largest firms. This phenomenon has created serious concerns over the role of smaller economic firms, of national businessmen and over the ultimate stability of the world economy. I believe that the most important forces behind the massive M&A wave are the same that underlie the globalization process: falling transportation and communication costs, lower trade and investment barriers and enlarged markets that require enlarged operations capable of meeting customers' demands. All these are beneficial, not detrimental, to consumers. As productivity grows, the world's wealth increases. Examples of benefits or costs of the current concentration wave are scanty. Yet it is hard to imagine that the merger of a few oil firms today could re-create the same threats to competition that were feared nearly a century ago in the U.S., when the Standard Oil trust was broken up. The mergers of telecom companies, such as WorldCom, hardly seem to bring higher prices for consumers or a reduction in the pace of technical progress. On the contrary, the price of communications is coming down fast. In cars, too, concentration is increasing-witness Daimler and Chrysler, Renault and Nissan-but it does not appear that consumers are being hurt. Yet the fact remains that the merger movement must be watched. A few weeks ago, Alan Greenspan warned against the megamergers in the banking industry. Who is going to supervise, regulate and operate as lender of last resort with the gigantic banks that are being created? Won't multinationals shift production from one place to another when a nation gets too strict about infringements to fair competition? And should one country take upon itself the role of “defending competition” on issues that affect many other nations, as in the U S. vs. Microsoft case ? 33. What is the typical trend of businesses today? [A] to take in more foreign funds. [B] to invest more abroad. [C] to combine and become bigger. [D] to trade with more countries. 34. According to the author, one of the driving forces behind M&A wave is ______ [A] the greater customer demands. [B] a surplus supply for the market. [C] a growing productivity. [D] the increase of the world's wealth. 35. From paragraph 4 we can infer that ______. [A] the increasing concentration is certain to hurt consumers [B] WorldCom serves as a good example of both benefits and costs [C] the costs of the globalization process are enormous [D] the Standard Oil trust might have threatened competition 36. Toward the new business wave, the writer's attitude can he said to be _______. [A] optimistic [B] objective [C] pessimistic [D] biased Passage 5 When I decided to quit my full time employment it never occurred to me that I might become a part of a new international trend. A lateral move that hurt my pride and blocked my professional progress prompted me to abandon my relatively high profile career although, in the manner of a disgraced government minister, I covered my exit by claiming “I wanted to spend more time with my family”. Curiously, some two-and-a-half years and two novels later, my experiment in what the Americans term “downshifting” has turned my tired excuse into an absolute reality. I have been transformed from a passionate advocate of the philosophy of “having it all”, preached by Linda Kelsey for the past seven years in the pages of She magazine, into a woman who is happy to settle for a bit of everything. I have discovered, as perhaps Kelsey will after her much-publicized resignation from the editorship of She after a build-up of stress, that abandoning the doctrine of “juggling your life”, and making the alternative move into “downshifting” brings with it far greater rewards than financial success and social status. Nothing could persuade me to return to the kind of life Kelsey used to advocate and I once enjoyed: 12-hour working days, pressured deadlines, the fearful strain of office politics and the limitations of being a parent on “quality time”. In America, the move away from juggling to a simpler, less materialistic lifestyle is a well-established trend. Downshifting-also known in America as “voluntary simplicity” has, ironically, even bred a new area of what might be termed anti-consumerism. There are a number of bestselling downshifting self-help books for people who want to simplify their lives; there are newsletter's, such as The Tightwad Gazette, that give hundreds of thousands of Americans useful tips on anything from recycling their cling-film to making their own soap; there are even support groups for those who want to achieve the mid- '90s equivalent of dropping out. While in America the trend started as a reaction to the economic decline——after the mass redundancies caused by downsizing in the late?80s——and is still linked to the politics of thrift, in Britain, at least among the middle-class downshifters of my acquaintance, we have different reasons for seeking to simplify our lives. For the women of my generation who were urged to keep juggling through the?80s, downshifting in the mid-'90s is not so much a search for the mythical good life——growing your own organic vegetables, and risking turning into one——as a personal recognition of your limitations. 37. Which of the following is true according to paragraph 1? [A] Full-time employment is a new international trend. [B] The writer was compelled by circumstances to leave her job. [C] “A lateral move” means stepping out of full-time employment. [D] The writer was only too eager to spend more time with her family. 38. The writer?s experiment shows that downshifting ____ [A] enables her to realize her dream [B] helps her mold a new philosophy of life [C] prompts her to abandon her high social status [D] leads her to accept the doctrine of She magazine 39. “Juggling one?s life” probably means living a life characterized by_____. [A] non-materialistic lifestyle [B] a bit of everything [C] extreme stress [D] anti-consumerism 40. According to the passage, downshifting emerged in the U.S. as a result of _____ [A] the quick pace of modern life [B] man?s adventurous spirit [C] man?s search for mythical experiences [D] the economic situation Part III English-Chinese Translation Directions: Read the following text carefully and then translate the underlined segments into Chinese. Your translation should be written clearly on ANSWER SHEET 2. (15 points) In less than 30 years? time the Star Trek holodeck will be a reality. Direct links between the brain?s nervous system and a computer will also create full sensory virtual environments, allowing virtual vacations like those in the film Total Recall. 41)There will be television chat shows hosted by robots, and cars with pollution monitors that will disable them when they offend. 42)Children will play with dolls equipped with personality chips, computers with in-built personalities will be regarded as workmates rather than tools, relaxation will be in front of smell television, and digital age will have arrived. According to BT?s futurologist, Ian Pearson, these are among the developments scheduled for the first few decades of the new millennium(a period of 1,000 years), when supercomputers will dramatically accelerate progress in all areas of life. 43)Pearson has pieced together the work of hundreds of researchers around the world to produce a unique millennium technology calendar that gives the latest dates when we can expect hundreds of key breakthroughs and discoveries to take place. Some of the biggest developments will be in medicine, including an extended life expectancy and dozens of artificial organs coming into use between now and 2040. Pearson also predicts a breakthrough in computer-human links. “By linking directly to our nervous system, computers could pick up what we feel and, hopefully, simulate feeling too so that we can start to develop full sensory environments, rather like the holidays in Total Recall or the Star Trek holodeck, ” he says. 44)But that, Pearson points out, is only the start of man-machine integration: “It will be the beginning of the long process of integration that will ultimately lead to a fully electronic human before the end of the next century.” Through his research, Pearson is able to put dates to most of the breakthroughs that can be predicted. However, there are still no forecasts for when faster-than-light travel will be available, or when human cloning will be perfected, or when time travel will be possible. But he does expect social problems as a result of technological advances. A boom in neighborhood surveillance cameras will, for example, cause problems in 2010, while the arrival of synthetic lifelike robots will mean people may not be able to distinguish between their human friends and the droids. 45)And home appliances will also become so smart that controlling and operating them will result in the breakout of a new psychological disorder—kitchen rage. Section V Writing 46. Directions: Among all the worthy feelings of mankind, love is probably the noblest, but everyone has his/her own understanding of it. There has been a discussion recently on the issue in a newspaper. Write an essay to the newspaper to 1)show your understanding of the symbolic meaning of the picture below. 2)give a specific example, and 3)give your suggestion as to the best way to show love. 第四部分 写作试题解析 一、审题谋篇 本题命题形式为漫画加提纲式写作。 提纲有三点要求:,1,描述画面的寓意,此处考生要注意symbolic meaning,比喻义,,(2)举一个具体的 例子对主题加以论证,(3)对于如何献爱心给出你的建议。 首先,审题时考生要抓住关键词。漫画加提纲式写作中,漫画中的字,特别是漫画的题目可谓是一字千 金,千万不可放过。本题的漫画中出现“爱心是一盏灯,在越黑暗的地方越明亮”一句话,该句最突出的词是“爱 心”,后半句话指出最需要爱心的环境——“黑暗中”,也就是说别人有困难的地方。因此漫画旨在提倡为需要 帮助的人或地方献出爱心。 其次,考生需要看清题目要求。要求指出文章应点明漫画所示内涵,并应举出具体事例,在此前提下提 出自己对献爱心的建议。由此可见,文章中既要联系社会举例,又要给出自己的评论,所以命题所测试的中 心内容为描述加议论。 考生可以在坚守三段论基础上采用叙议结合的方法:第一段需围绕图画展开,将图画中所有信息描述清 楚,点出漫画所示内涵。第二段要针对主题举具体的例子,由本题中心可知,举的例子应体现两要素:“爱心” 和“需要”。第三段应在联系实际,突出中心的基础上,提出具体建议。 二、参考范文 As can be seen from this vivid picture, like a light, love means much more to those who are in great difficulty than to those who live in comforts. Love, as illustrated in the picture,is just like the oil lamp,which certainly shines brighter in the dark. In other words,the darker the place is,the more precious the light will be. Love is somewhat the same: it is most needed by those who are in urgent need of help and considered most precious in most difficult situations. Therefore, as people living in the modern society, we all should contribute our love to those needy people. I can exemplify this conclusion with the Poverty Alleviation Project. It is known to all that the Chinese government has been calling for people participating in the project. Obviously, the expression of love can be best demonstrated by helping the poor in some backward regions. Therefore, many college students choose to work in these regions upon graduation. It is in these places that they are most needed and their knowledge can be made best use of. The best way to show our love,in my opinion,is to follow the above mentioned example, giving love to the people during the hours of darkness. So when we see someone in difficulty or in distress,don?t hesitate to offer our hands. I believe the relationship between people will be more harmonious and our society will be a better place for us to live in. Let?s do as a famous saying goes:Ask not what others can do for you; ask what you can do for others. 三、范文点评 文章结构: 本文思路清晰,重点突出。首段开篇点题,接着,考生清晰地阐述了灯光的象征意义,段尾再次概括全 文的主题,段落几个部分相互呼应。第二段中,考生举出一个既有深度,崇高性,,又有广度,社会性,的例 子——扶贫工程,并在段尾总结论证论点:要在别人困难的时候献爱心。第三段,作者提出具体建议,最后 提出号召,并以流行语点睛。 语言亮点: 1. vivid:形象的,生动的。副词为vividly。 2. in great difficulty和in comforts:两个介词结构意思相反,形成对仗。 3. illustrate:,用插图或图画,说明,阐明。例如:The cartoon illustrates a profound principle in life.,这幅 画揭示了一个深刻的人生哲理, 4. the darker …the more precious…:这里使用的是the more … the more …结构,例如:The faster our country?s industry develops, the more efforts we should spare in protecting the environment.,工业发展越快,我们 越应该花精力保护环境,。 5. precious:珍贵的,宝贵的。如:Time is precious.,时间很宝贵,。 6. contribute:对…做贡献,名词形式是contribution,如:make a contribution。 7. exemplify:用例子说明,例证。例如:This story exemplifies what I just said.,这个故事正好说明了我刚 才说的话,。类似用法有:illustrate, serve as an example。 8. participate in:参加,类似的用法有:take part, join in, partake, get involved。 9. demonstrate:表现,表明,证明。例如:This cartoon demonstrates a widespread phenomenon in society. ,这幅画表现了社会上的一个普遍现象,。 10. upon graduation:upon后加名词,可表示“…的时候”。如:Upon seeing their child make achievements, the parents are always as joyful as anything.,当看到自己的孩子取得成绩,父母总是高兴得不得了, 11. It is…that,who,…:常引导结构强调句,it is后接句子中被强调的部分,它可以是状语、主语、宾语或宾与补足语,但不可以是表语、谓语动词和让步状语从句或对比状语从句。若强调的是人,用关系代词who,其它都用that。如:It is with the development of technology and population growth, the amount and range of materials used has increased at an alarming rate.,正是随着技术的发展和人口的增长,使用材料的数量和范围也都在以惊人的速度增长,。在写作中让句型保持多样化可以给文章增加不少色彩和可读性,强调句就是一个很好的选择,它使句子简洁而有力度。 12. follow the above mentioned example:follow example表示“模仿…”,above mentioned的意思是“上面提及的”。 13. offer our hands:“帮助”,类似的表达有:help out, extend a hand, lend a hand, give assistance。 14. harmonious:“和睦的”,该词常形容family或relations。 15. Ask not what others can do for you; ask what you can do for others:不要问别人能为你做什么,而问你能为别人做什么。该句来自于美国总统肯尼迪的名言:“Ask not what your nation can do for you; ask what you can do for your nation.”,不要问你的国家能为你做什么,而要问你能为你的国家做什么,。能够在作文中恰当地用上名言的考生恐怕不会太多,但是一旦用上就会引起阅卷老师的注意,得高分的可能性也就越大。因此考生平时可注意加强这方面的准备。如:The people who get on in this world are the people who get up and look for circumstances they want, and if they cannot find them, make them.,在这个世界上取得成功的人,都努力去寻找他们想要的机会,如果找不到时,他们就自己创造机会。——英国著名作家萧伯纳, 四、写作误区 篇章结构误区: 首先,考生应当看清题目要求。一看到油灯,有的考生就按照固定思维,想当然地认为文章中心是赞扬“春蚕到死丝方尽,蜡炬成灰泪始干”的人民教师,结果写成了一篇教师颂。指导语中虽说明了“Everyone has his/her own understanding of love”,但图画上已经明确给出了主题:“爱心是一盏灯,在越黑暗的地方越明亮”,也就是说爱心在最需要它的地方显得最有价值。而有的考生花了大量笔墨赞美爱,包括爱情、友情、亲情等,论述“love is noblest feeling”,有的考生则联想到社会上的黑暗面,对缺乏“爱心”的人进行一番批判。以上都是由于审题不慎而造成的跑题。其次作文要求中规定要举一个例子说明问题。因此,考生所举的例子应当具体并且具有代表性。不可泛泛而谈,也不可虚构一些不切实际的例子。本文要想写得有深度,很大程度上由例子的深度和广度决定。因此,要想得高分,例子至关重要,应具有广阔的社会性,如“希望工程”、“长江抗洪”、“扶贫计划”、帮助下岗职工再就业等等。 语言表达错误: ?词性错误: Our world will bright if everyone shows love to others.,Our world will become bright if everyone shows love to others., ?搭配不当: Although the light is weak, it plays a great effect which will make people more firmly than before.,Although the light is weak, it has a great effect which will make people more firm than before., ?代词不一致: When every one devotes some love to the world, you will find the world is brighter and more beautiful.,When every one gives his share of love to the world, he will find the world is brighter and more beautiful., ?介词错误: I was greatly worried my future life.,I was greatly worried about my future life., ?非谓语结构错误: The best way show love is to help those need help in their life.,The best way to show love is to help those needing / who need help in their life., ?累赘: For those people who still live very poor lives, the help from other people can give them light and hope to improve their lives.,Poor people can get light and hope from others? help to improve their lives., ?综合性错误: Even if the government gives more money to help children in poor areas, the rate of them are helped is still very low.,Even if the government appropriates more money to help children in poor areas, the number of those who can be helped is still very small .
本文档为【1994—2001年考研英语真题(附作文解析)1[新版]】,请使用软件OFFICE或WPS软件打开。作品中的文字与图均可以修改和编辑, 图片更改请在作品中右键图片并更换,文字修改请直接点击文字进行修改,也可以新增和删除文档中的内容。
该文档来自用户分享,如有侵权行为请发邮件ishare@vip.sina.com联系网站客服,我们会及时删除。
[版权声明] 本站所有资料为用户分享产生,若发现您的权利被侵害,请联系客服邮件isharekefu@iask.cn,我们尽快处理。
本作品所展示的图片、画像、字体、音乐的版权可能需版权方额外授权,请谨慎使用。
网站提供的党政主题相关内容(国旗、国徽、党徽..)目的在于配合国家政策宣传,仅限个人学习分享使用,禁止用于任何广告和商用目的。
下载需要: 免费 已有0 人下载
最新资料
资料动态
专题动态
is_105949
暂无简介~
格式:doc
大小:541KB
软件:Word
页数:178
分类:生活休闲
上传时间:2017-10-18
浏览量:40